ORTHOPEDIC MCQS BANK HAND AND WRIST 1C

 
  1. A 22-year-old college quarterback is tackled and sustains a reducible first carpometacarpal dislocation. What is the recommended treatment?

    1. Closed reduction and casting

    2. Closed reduction and percutaneous pinning

    3. First carpometacarpal arthrodesis

    4. Dorsal capsulodesis

    5. Ligament reconstruction using tendon autograft

    CORRECT ANSWER: 5

    When comparing closed reduction and pinning to ligament reconstruction, the reconstruction group had slightly better abduction and pinch strength. The volar oblique ligament usually tears off the first metacarpal in a subperiosteal fashion. In this young patient, motion-sparing procedures are preferred.

     

     

     

  2. Figures 14a and 14b show the clinical photographs of a patient who was stranded in a subzero region for several days. The photographs were taken the morning after arrival in the hospital. The patient is otherwise healthy and fit, and takes no medication. He has no clinical signs of sepsis. He reports burning pain and tingling in both feet. What is the best treatment?

     

     

     

     

     

     

    1. Moist dressings and continued observation

    2. Debridement of the necrotic tissue

    3. Amputation at the metatarsophalangeal level with open wound management

    4. Closed forefoot amputation

    5. Guillotine transtibial amputation

    CORRECT ANSWER: 1

    The patient has no clinical or observed signs of sepsis. The skin just proximal to the gangrenous tissue appears somewhat hyperemic and is clearly viable. These wounds should be managed much like burn wounds. Moist dressings should be used until the tissue clearly demarcates. Much of the insult may simply be superficial and only require late debridement.

     

     

     

  3. Which of the following conditions precludes performing a tendon transfer?

    1. The target joint has a full passive range of motion.

    2. The range of motion of the target joint only occurs in the direction of correction.

    3. The target joint cannot be passively corrected to its neutral position.

    4. The muscle to be transferred is out-of-phase.

    5. There is no pulley to assist the transferred muscle’s fulcrum.

     

    CorreCt answer: 3

     

    Several conditions must be met before a tendon transfer has the potential to correct a dynamic deformity. If the target joint cannot be passively corrected to neutral, it indicates that a static joint contracture or bony deformity exists that cannot be corrected with a dynamic tendon transfer. While in-phase muscles are best, out-of-phase muscles are often the only muscles available for transfer. Tendon transfer should pull in a straight line to avoid tethering and late failure.

     

     

     

  4. A 20-year-old football player presents with a one week history of right index finger pain which started after his hand got caught in a face mask during a tackle. Physical exam shows swelling of the digit with no breaks in the skin, and no active flexion. AP, lateral, and oblique radiographs are provided in Figures A, B, and C respectively. Which of the following structures most often prevents closed reduction of this injury?

     

     

     

     

     

     

    1. Volar plate

    2. Collateral ligaments

    3. FDP tendon

    4. Central slip

    5. Dorsal capsule CORRECT ANSWER: 1

    Figures A, B, and C demonstrate a dorsal dislocation of the DIP joint without associated fracture.

     

    Abouzahr et al conducted a case report and literature review on irreducible dorsal DIP dislocations. The authors found that the most common block to reduction with a closed injury is an interposed volar plate, which is avulsed from its origin on the middle phalanx. They recommened open reduction and extraction of the volar plate if one is unable to achieve concentric stable reduction after two attempts. Furthermore, the authors determined that in open injuries, the FDP tendon is primarily responsible for irreducibility. The collateral ligaments are less likely to be involved in this case because there is little coronal deformity present. The dorsal capsule is typically not a block to reduction, and the central slip is disrupted (but does not block reduction) in volar PIP joint dislocations.

     

     

     

  5. A 52-year-old woman slips in her bathroom and strikes her right hand on a cabinet. She notes swelling, ecchymosis, and pain with attempted motion. There are no open wounds. Radiographs are shown in Figures 5a through 5c. What is the most appropriate treatment?

     

     

     

     

     

     

     

     

     

    1. Immobilization of the hand with the metacarpophalangeal (MCP) joints in flexion

    2. Immobilization of the hand with the MCP joints in extension and the IP joints in extension

    3. Percutaneous pinning of the proximal phalanx

    4. Open reduction and internal fixation of the proximal phalanx

    5. Early motion and pain management

    CORRECT ANSWER: 1

    Nondisplaced transverse fractures of the phalanges are stable. Immobilization in the intrinsic plus position will prevent MCP joint stiffness. Displaced oblique fractures are more at risk for instability.

     

     

  6. A 19-year-old college student reports a 1-week history of wrist pain following an intramural rugby match. A PA radiograph is shown in Figure 6. He denies any prior wrist injury. What is the best course of action?

     

     

     

     

    1. Closed reduction and long arm cast immobilization

    2. Closed reduction and short arm cast immobilization

    3. Closed reduction and percutaneous pinning

    4. Open reduction and internal fixation with Kirschner wires

    5. Open reduction and internal fixation with a headless, cannulated compression screw

    CorreCt answer: 5

     

    The patient has a scaphoid fracture involving the proximal pole. Surgical treatment is recommended for such fractures because of the prolonged period of cast immobilization necessary and the increased risk of delayed union, nonunion, and/or osteonecrosis with nonsurgical management. A cannulated compression screw, inserted in the central scaphoid via a dorsal approach, is biomechanically advantageous and provides greater stability for fracture healing than Kirschner wires. Recently, good outcomes have been reported with arthroscopic-assisted percutaneous fixation of nondisplaced or minimally displaced scaphoid fractures.

     

     

  7. A 47-year-old woman falls and sustains a direct blow to her middle finger. She notes pain and swelling and is unable to move the proximal interphalangeal (PIP) or distal interphalangeal (DIP) joints. Radiographs are shown in Figures 8a through 8c. Proper management should consist of

     

     

     

     

     

     

     

    1. closed reduction and splinting in metacarpophalangeal (MCP) and PIP joint extension.

    2. closed reduction and splinting in MCP joint flexion and PIP joint extension.

    3. reduction and percutaneous intramedullary Kirschner wire fixation.

    4. reduction and lag screw fixation.

    5. buddy taping and early range of motion.

     

    CorreCt answer: 4

     

    The oblique nature of the fracture and extension of the fracture to the condyles implies an unstable fracture. Lag screw fixation provides an excellent chance of union, and the ability to start early range of motion.

     

     

     

  8. A 19-year-old collegiate baseball player injures the ring finger on his dominant hand while sliding headfirst into second base. He reports that he is unable to actively flex or extend the distal interphalangeal joint of the finger. Radiographs are shown in Figures 19a and 19b. What is the anatomic lesion leading to this injury?

     

     

     

     

     

     

    1. Rupture of the terminal extensor tendon

    2. Avulsion of the volar plate

    3. Rupture of the sagittal bands

    4. Rupture of the spiral oblique retinacular ligaments

    5. Rupture of the profundus insertion

    CORRECT ANSWER: 5

    The radiographs reveal a bony avulsion of the flexor profundus insertion (Jersey finger). The large bony fragment classifies this as a Leddy type III injury. The bony fragment has retracted to the level of the annular pulley (A4).

     

     

     

  9. A 72-year-old man was involved in an automobile accident 4 weeks ago. Initially he noted pain about his nondominant left

    shoulder, which resolved within a few weeks after the accident. He now describes trouble with gripping and carrying items in his left hand. Radiographs are shown in Figures 20a through 20c. His signs and symptoms are the result of injury to which of the following ligaments?

     

     

     

     

     

     

     

    1. Volar scapholunate interosseous

    2. Dorsal scapholunate interosseous

    3. Long radiolunate

    4. Short radiolunate

    5. Dorsal intercarpal CORRECT ANSWER: 2

    The radiographs reveal a gap between the scaphoid and lunate bones, indicative of disruption of the scapholunate ligament complex. The three

    components of the complex are the dorsal scapholunate ligament, the volar (or palmar) scapholunate ligament, and the proximal fibrocartilaginous membrane, listed in decreasing yield strength. Disruption of the stout dorsal interosseous ligament is required for scapholunate dissociation to occur.

     

     

     

  10. A 21-year-old college student fell from a balcony and landed on his outstretched right hand. He is seen in the emergency department 4 hours later and reports wrist pain and diffuse hand numbness. The volar forearm compartment is soft and there is no pain with passive finger extension. Radiographs are shown in Figures 25a and 25b. Definitive treatment should consist of

     

     

     

     

     

     

    1. closed reduction and cast immobilization.

    2. closed reduction, carpal tunnel release, and cast immobilization.

    3. open reduction and ligament repair via a dorsal approach.

    4. open reduction and ligament repair via a volar approach.

    5. open reduction and ligament repair via dorsal and volar approaches and an extended carpal tunnel release.

    CorreCt answer: 5

     

    A spectrum of perilunate injury patterns exists, with the dorsal trans-scaphoid perilunate fracture-dislocation being the most common. Perilunate injuries are highly unstable complex carpal disruptions that are not amenable to closed treatment. Open reduction and internal fixation is necessary to accurately restore carpal alignment via fracture reduction and fixation and intercarpal ligament repair. Controversy exists regarding the need for dorsal or combined dorsal and volar approaches. Based on the radiographic findings of a volar dislocation of the lunate and the associated median nerve injury, the patient requires open reduction and internal fixation via combined dorsal and volar approaches with a concomitant carpal tunnel release.

     

     

     

  11. A 69-year-old man sustains a traumatic amputation to the distal phalanx of his little finger while working with power tools. Radiographs are shown in Figures 27a and 27b. The patient was instructed how to perform wet-to-dry dressing changes in the emergency department. Clinical pictures taken in the office are shown in Figures 27c through 27e. What is the most appropriate management of this soft-tissue wound?

     

     

     

     

     

     

     

     

     

     

     

     

     

    1. Continue wet-to-dry dressing changes until the wound heals by secondary intention.

    2. Perform a volar advancement flap (ie, Moberg flap).

    3. Perform a V-Y advancement flap.

    4. Perform a thenar flap.

    5. Perform a cross-finger flap to the ring finger.

     

    CorreCt answer: 1

     

    The clinical photographs and radiographs reveal a distal phalangeal amputation with soft-tissue coverage over nonexposed bone. This is an ideal circumstance to allow healing by secondary intention with wet-to-dry dressing changes. There are few complications and the aesthetics surpass that of any soft-tissue reconstruction procedure. Volar advancement flaps (Moberg flaps) are limited to small defects about the thumb. A thenar flap will provide good coverage; however, the results are not comparable to simple dressing changes. A V-Y flap is useful when there is more tissue loss dorsally.

     

     

     

  12. A 42-year-old college professor reports persistent pain at the hypothenar eminence 9 months after falling from his bicycle. Initial radiographs were reportedly normal. Use of a wrist splint for the last 2 months has failed to provide relief. A radiograph obtained by his primary care physician prior to referral is seen in Figure 31. What is the most appropriate treatment?

     

     

     

    1. Continued splint immobilization for 6 weeks

    2. Pisiform excision

    3. Hook of hamate excision

    4. Open reduction and internal fixation with bone grafting

    5. Lunotriquetral arthrodesis

    CORRECT ANSWER: 2

    The oblique radiograph of the wrist reveals a displaced fracture of the pisiform that usually occurs following a direct blow or sudden contraction of the flexor carpi ulnaris tendon. Persistent pain secondary to pisotriquetral incongruity and degenerative arthritis warrants pisiform excision, which does not compromise wrist flexion strength. Pisiform fractures are usually missed on routine radiographic views. An oblique or carpal tunnel view can be helpful in visualizing the pisotriquetral joint.

     

     

     

  13. A 30-year-old woman injured the ring finger of her nondominant hand while playing baseball 5 weeks ago. She now reports pain and limited motion of the proximal interphalangeal (PIP) joint. A lateral fluoroscopy image is shown in Figure 36. Treatment of the PIP joint should consist of

     

     

     

    1. closed reduction and percutaneous pinning.

    2. implant arthroplasty.

    3. arthrodesis.

    4. volar plate arthroplasty.

    5. resection arthroplasty.

     

    CorreCt answer: 4

     

    The patient has a neglected PIP joint fracture-dislocation with comminution involving more than 40% of the volar articular surface of the middle phalanx. Volar plate arthroplasty has been advocated for the treatment of acute unstable and chronic dorsal fracture-dislocations. The volar plate is incised laterally and released from the collateral ligaments. The volar fragments of the middle phalanx are removed and a trough is created for advancement of the volar plate, which is secured with sutures secured on the dorsum of the middle phalanx beneath the extensor mechanism.

     

     

     

  14. A 19-year-old woman fell onto her nondominant hand 6 weeks ago. Radiographs are shown in Figures 37a and 37b. A decision has been made to treat this fracture surgically. What is the best approach to treat this fracture?

     

     

     

     

     

     

    1. Percutaneous pinning of the fracture with Kirschner wires

    2. Open reduction and pinning with Kirschner wires

    3. Arthroscopic in situ compression screw fixation

    4. Reduction and compression screw fixation via a volar approach

    5. Reduction and compression screw fixation via a dorsal approach

    CORRECT ANSWER: 5

    Displaced fractures of the scaphoid are best treated with compression screw fixation. Proximal third fractures (as in this patient) are optimally approached via a dorsal approach to ensure proper reduction and compression. Fractures of the scaphoid waist can be approached either by a volar or a dorsal approach. Kirschner wire fixation is limited to proximal pole fractures that are too small to accommodate the trailing head of a compression screw.

     

     

     

  15. In the majority of patients, the deep palmar arch has a main contribution from what vessel which travels between the two heads of the first dorsal interosseous muscle?

    1. Superficial palmar branch of the radial artery

    2. Princeps pollicis artery

    3. Deep dorsal (terminal) branch of the radial artery

    4. Common palmar digital artery

    5. Ulnar artery CORRECT ANSWER: 3

    The deep palmar arch (deep volar arch) is an arterial network found in the palm. In the majority of patients it is formed mainly from the terminal part of the radial artery, with the ulnar artery contributing via its deep palmar branch. This is in contrast to the superficial palmar arch, which is formed predominantly by the ulnar artery with some contribution by the superficial palmar branch of the radial artery. The deep palmar arch lies upon the bases of the metacarpal bones and on the interossei of the hand, being covered by the oblique head of the adductor pollicis muscle, the flexor tendons of the fingers, and the lumbricals of the hand. At the wrist the radial artery passes between the radial collateral ligament and the first dorsal compartment after which it dives between the heads of the first dorsal interossei where it anastomoses with the deep palmar branch from the ulnar artery, completing the deep volar arch.

     

     

     

     

     

  16. A butcher sustains a traumatic amputation of the ring finger through the distal interphalangeal joint. He is brought to the operating room where the flexor digitorum tendon is retrieved and advanced to the distal stump. Three months after surgery the patient notes that when he makes a fist, only his ring finger tip reaches the palm. What is this patient's clinical problem?

    1. mallet finger

    2. swan neck deformity

    3. boutonniere deformity

    4. lumbrical plus deformity

    5. quadrigia syndrome CORRECT ANSWER: 5

    Quadrigia syndrome occurs when a flexor digitorum profundus (FDP) tendon is shortened and advanced. Malerich et al found the tolerable degree of FDP advancement was 1 cm. The common muscle belly of the FDP prevents the tendons to the other fingers from reaching full excursion. Treatment is release of the shortened tendon. Lumbrical plus occurs when the FDP tendon retracts and causes paradoxical interphalangeal extension when trying to flex. Mallet

    finger is an injury to the terminal extensor tendon. Boutonniere deformity occurs from central slip injury and results in PIP flexion and DIP extension. Swan-neck deformity consists of hyperextension at the PIP joint with flexion at the DIP joint typically caused by volar plate attenuation.

     

     

     

  17. A 30-year-old man caught his dominant little finger on the straps of his windsurfing board 10 days ago. He reports swelling about the distal phalanx and has difficulty completely extending the distal interphalangeal joint. A radiograph is shown in Figure 47. What is the most appropriate treatment for this injury?

     

     

     

     

    1. Extension splinting of the proximal interphalangeal and distal interphalangeal joints

    2. Extension splinting of the distal interphalangeal joint

    3. Transarticular pinning of the distal interphalangeal joint

    4. Extension block pinning of the distal interphalangeal joint

    5. Open reduction and internal fixation of the distal phalanx

    CORRECT ANSWER: 2

    The radiograph reveals a “bony mallet injury.” As the distal phalanx is not volarly subluxated, extension splinting, similar to a classic mallet injury without bony involvement, is appropriate. If there is volar subluxation associated with a large bony fragment, surgical intervention is appropriate.

     

     

     

  18. A 38-year-old man caught his index finger in a volleyball net. He noted an angular deformity of the finger that was reduced when a teammate pulled on his finger. Three weeks later, he now reports trouble extending his finger. A clinical photograph is shown in Figure

    55. What anatomic structure is most likely injured?

     

     

     

    1. Spiral oblique retinacular ligament

    2. Sagittal bands

    3. Volar plate

    4. Central slip of the extensor tendon

    5. Terminal extensor tendon

    CORRECT ANSWER:

    4

    The clinical photograph shows a classic boutonniere deformity. It is likely that the patient sustained a volar dislocation of the proximal interphalangeal joint, with a concomitant rupture of the central slip insertion of the extensor tendon.

     

     

     

  19. A 66-year-old woman was a restrained passenger in an automobile accident. She sustained a direct blow to her nondominant left hand as the airbag in her automobile deployed and she now reports pain, swelling, and difficulty moving her fingers. Radiographs are shown in Figures 58a and 58b. Appropriate definitive treatment should consist of

     

     

     

     

     

     

    1. application of a compressive soft dressing, and aggressive edema control and range-of-motion exercises.

    2. plaster immobilization without reduction of the fractures.

    3. closed reduction and plaster immobilization.

    4. surgical fixation of the middle finger metacarpal and closed treatment of the ring and little finger metacarpals.

    5. surgical fixation of all three metacarpal fractures.

     

    CorreCt answer: 5

     

    While most isolated metacarpal fractures can be treated nonsurgically, multiple metacarpal fractures are inherently unstable due to the loss of support that an intact adjacent metacarpal provides; therefore, treatment should consist of surgical fixation of all three metacarpal fractures.

     

     

  20. A 54-year-old gentleman presents to your office with a mass on top of the distal phalanx that has enlarged over the last nine months. His main complaint is significant tenderness to palpation over the mass. There is no pain with forcible movement of his fingers. A clinical photo is shown in Figure A. A dedicated radiograph of the distal phalanx is shown in Figure B. What treatment option is most appropriate for the best patient outcome?

     

     

     

     

     

     

     

    1. Observe

    2. Needle aspiration

    3. Fusion of distal interphalangeal joint

    4. Removal of bone spur and cyst

    5. Obtain infectious work-up

    CORRECT ANSWER:

    4

    This is a classic presentation of a mucous cyst. The most appropriate treatment would be excision of the cyst and removal of the underlying bone spur.

    A mucous cyst of the hand is usually a small, soft, benign structure. They are associated with osteoarthritis and develop around bone spurs near a joint.

    Surgery is typically recommended if there is significant pain at the site of the cyst or with range of motion of the involved joint. Nail bed deformity may occur with disease progression if left untreated.

     

    Rizzo et al. examined a series of 132 patients with mucous cysts, comparing outcomes between injection and surgery. They found that 60% of people with aspiration and steroid injection had complete resolution of the cyst compared to 100% with excision.

     

    Figure A shows a small mucous cyst just proximal to the nailbed. Figure B shows a radiograph of the distal interphalangeal joint. There is extensive joint arthritis with dorsal bone spurs.

     

    Incorrect Answers:

    Answer A: Pain from a mucous cyst is usually constant, but in some people it may come and go. Rarely the cysts will resolve over time. Typically, the mucous cyst will progress with time and cause nail deformity.

    Answer B: Aspiration will lead to a >40% recurrence.

    Answer C: Removal of cyst and joint fusion would be indicated if there was pain with with any forcible movement of the joint.

    Answer E: Infection is usually not associated with a benign mucous cyst.

     

     

     

  21. Conduction velocity changes in peripheral nerve chronic compression syndromes result primarily from which of the following actions?

    1. Wallerian degeneration distal to the compression

    2. Wallerian degeneration proximal to the compression

    3. Transection of individual axons

    4. Schwann cell proliferation and apoptosis

    5. Apoptosis of the neuronal cell body

    CORRECT ANSWER: 4

    Whereas early literature proposed Wallerian degeneration, recent studies have found that chronic compression of the peripheral nerve induces Schwann cell turnover, including both proliferation and apoptosis without axonal pathology. Additionally, the Schwann cell phenotype becomes less promyelogenic and upregulates proregenerative molecules such as vascular endothelial growth factor (VEGF). Wallerian degeneration is a process that results from separating

    the axon from the cell body and the hallmark is granular disintegration of the axonal cytoskeleton that is triggered by increased axoplasmic calcium.

     

     

     

  22. All of the following can be found on the electromyography (EMG) portion of an electrodiagnostic study during the evaluation of a patient with carpal tunnel syndrome EXCEPT:

    1. Fibrillations at rest

    2. Positive sharp waves

    3. Decreased motor recruitment

    4. Increased insertional activity

    5. Increased distal sensory latency

      CORRECT ANSWER: 5

      EMG's detect the electrical potential generated by muscle cells when these cells are electrically activated. They give information about the muscle motor unit and can display the presence of fibrillations, sharp waves, motor recruitment, and insertional activity of the muscle. The nerve conduction (NCV) portion of the electrodiagnostic study measures the speed at which the nerve impulse travels down the axon. Large, myelinated nerve fibers conduct impulses the fastest and thus only these fibers are evaluated in the nerve conduction portion of the electrodiagnostic study. Distal latencies and conduction velocities are measured with NCV's. General parameters for NCV diagnosis of carpal tunnel syndrome include a distal motor latency of >4.5 msec, a distal sensory latency of >3.5msec, or a conduction velocity of < 52 m/sec.

       

      The articles by Brumback et al and Gooch et al is a review of electrodiagnostic studies for compression neuropathies.

       

       

       

  23. Pediatric flexor tendon injuries of the upper extremity differ from adult flexor tendon injuries in which of the following ways?

    1. Delayed presentation is not common.

    2. A staged repair is never necessary.

    3. Six to eight weeks of postoperative immobilization is recommended.

    4. Cooperation with occupational therapy can be difficult.

    5. The use of Botulinum is contraindicated.

    CorreCt answer: 4

     

    Pediatric flexor tendon injuries have several remarkable distinctions from those in adults. Delayed presentation is more common in children, at times requiring staged flexor tendon reconstruction. Three to four weeks of postoperative immobilization following acute repair is recommended in children as opposed to early motion protocols used in adults. Temporary paralytic agents (botulinum toxin type A) have also been shown to facilitate the rehabilitation phase of flexor tendon care in very young children.

     

     

     

  24. A 34-year-old seamstress was diagnosed with Parsonage-Turner brachial neuritis in the right upper extremity 1 month ago. She has weak forearm pronation with the elbow in the flexed position. She denies any current sensory changes. A clinical image of her hands attempting to make a clenched fist are shown in Figure A. Which of the following most likely represents her diagnosis and treatment?

     

     

     

     

    1. Anterior interosseous nerve syndrome treated with observation

    2. Posterior interosseous nerve syndrome with release of the Arcade of Frohse

    3. Pronator syndrome with surgical release of the lacertus fibrosis

    4. Anterior interosseous nerve syndrome with surgical release of Gantzer's muscle

    5. Posterior interosseous nerve syndrome treated with observation

    CORRECT ANSWER: 1

    This patient presents with anterior interosseous nerve (AIN) syndrome and is often seen in conjunction with brachial neuritis (Parsonage-Turner Syndrome).

    AIN syndrome leads to motor palsies of the flexor pollicis longus and the two radial profundus tendons leading to the clincal image shown in Figure A. The pronator quadratus is also involved and can be tested with the elbow held in a flexed position to neutralize the humeral head of the pronator teres muscle.

    No sensory changes occur and electromyographic (EMG) and nerveconduction (NCV)studies are often helpful in establishing the diagnosis.

     

    The Level 5 review article by Rodner et al. discusses that anterior interosseous nerve syndrome usually resolves with time, particularly if the lesion is secondary to neuritis. Observation for 3 to 6 months with splinting at 90 degrees is favored before surgical treatment.

     

    Parsonage and Turner reported in their 1948 landmark article, on several cases of isolated AIN palsy caused by neuralgic amyotrophy (ie, Parsonage-Turner syndrome [PTS] or brachial plexus neuritis). PTS symptoms may include pain and motor and/or sensory dysfunction in one or in multiple peripheral nerves of the upper extremity.

     

     

     

  25. A 23-year-old national team rower reports pain over the radial dorsum of the forearm that is made worse with flexion and extension of the wrist during competition. His primary physician initially diagnosed de Quervain's tenosynovitis, and a subsequent corticosteroid injection into the first dorsal compartment at the wrist provided no relief. The patient continues to report pain and audible crepitus that is noted 5 cm proximal to the wrist joint, on the radial aspect. What structures are involved in the continued pathology?

    1. Abductor pollicis brevis and extensor pollicis brevis

    2. Abductor pollicis brevis and extensor pollicis longus

    3. Abductor pollicis longus and extensor pollicis brevis

    4. Abductor pollicis longus and extensor pollicis longus

    5. Adductor pollicis and extensor pollicis longus

    CORRECT ANSWER: 3

    Intersection syndrome is also known as "squeakers wrist," "oarsmen wrist," and crossover tendinitis. It occurs where the first and second dorsal wrist compartment structures pass over one another, resulting in fibrosis, muscular changes, and inflammation of the bursa in this area. The structures involved are the abductor pollicis longus and extensor pollicis brevis (first dorsal compartment) that pass across the second compartment structures (extensor carpi radialis brevis and extensor carpi radialis longus). An audible "squeak" is

    occasionally heard at the intersection point, which is approximately 4 to 5 cm proximal to the proximal dorsal wrist crease.

     

     

     

  26. A 20-year-old male tennis player reports the acute onset of ulnar-sided wrist pain after hitting a forehand shot. Examination reveals dorsoulnar tenderness and minimal swelling. The pain is recreated with supination, wrist flexion, and ulnar deviation. Radiographs are normal. What structure is most likely involved?

    1. Ulnar styloid

    2. Flexor carpi radialis tendon

    3. Extensor carpi ulnaris tendon

    4. Scapholunate ligament

    5. Transverse carpal ligament

    CORRECT ANSWER: 3

    Extensor carpi ulnaris (ECU) lesions produce pain at the dorsoulnar aspect of the wrist, particularly during wrist supination, wrist flexion, and ulnar deviation. It has been frequently described in tennis players. Most ECU tenosynovitis can be successfully treated nonsurgically with immobilization techniques. Surgical treatment is generally indicated for ECU tenosynovitis or tendinopathy that does not respond to rest. Anatomically, the ECU retinaculum can rupture and the tendon can leave its sheath. With supination, the tendon can leave the sheath and then return to its position during pronation.

     

     

     

  27. A 30-year-old female reports 5 months of wrist pain after a fall onto her wrist. A radiograph is shown in Figure A. If untreated, which of the following is least likely to occur during the natural progression of the disease process?

     

     

     

    1. Radial styloid osteophyte

    2. Radioscaphoid arthritis

    3. Midcarpal arthritis

    4. Scaphocapitate arthritis

    5. Radiolunate arthritis CORRECT ANSWER: 5

    Radiographs show a scaphoid non-union which can lead to Scaphoid Nonunion Advanced Collapse (SNAC wrist) and progressive arthritis. The natural history of degenerative changes first occurs at the radioscaphoid area and progresses to pancarpal arthritis. All of the answers above are features of a SNAC wrist with radiolunate arthritis only occurring at the very end stages of disease.

     

    In the cited reference by Schuind et al, they found that professional heavy work, age of the nonunion of over 5 years, associated radial styloidectomy, and duration of postoperative immobilization were associated with a significantly decreased likelihood of healing of the scaphoid nonunion with operative treatment.

     

    The study by Soejima et al found that distal scaphoid resection produces a satisfactory clinical outcome and should be considered one of the surgical options for patients with long-standing scaphoid nonunion with either radioscaphoid or intercarpal degenerative arthritis.

     

     

  28. A 20-year-old man has a dorsal metacarpophalangeal dislocation of the index finger. Multiple attempts to reduce the dislocation in the emergency department have not been successful. What structure is most likely preventing the joint from being reduced?

    1. First dorsal interosseous

    2. Radial collateral ligament

    3. Ulnar collateral ligament

    4. Natatory ligament

    5. Volar plate CORRECT ANSWER: 5

    The volar plate is the structure that usually prevents the finger metacarpophalangeal joint from reducing. Blockage by the first dorsal interosseous is not a common reason for an irreducible metacarpophalangeal joint dislocation. None of the other structures commonly prevent metacarpophalangeal joint reduction.

     

     

     

  29. A 38-year-old man reports a 6-month history of pain in his left wrist. He denies any injury and is otherwise healthy. An MRI scan is shown in Figure 24. What is the recommended treatment?

     

     

     

    1. Radial shortening osteotomy

    2. Lunate excision with tendon interposition

    3. Lunate implant arthroplasty

    4. Ulnar shortening osteotomy

    5. Total wrist arthrodesis CORRECT ANSWER: 1

    The MRI scan shows avascularity (decreased signal intensity on T1-weighted image) of the lunate in an ulnar minus wrist, consistent with Kienbock's disease. No degenerative changes are seen in the carpus. Of the choices listed, radial shortening osteotomy is the treatment of choice. This procedure provides an extra-articular approach to treatment. The other options could be considered in more advanced cases or if joint deterioration/destruction was noted. Lunate excision with tendon interposition and lunate implant arthroplasty are rarely used at this time. An ulnar shortening osteotomy could make the problem worse by increasing the contact forces between the radius and lunate. A limited intercarpal fusion is usually used prior to resorting to total wrist arthrodesis.

     

     

     

  30. Normal thumb flexor tendon kinematics are restored by repairing which of the following pulleys when the A-2 is intact?

    1. Av-2

    2. Av-1

    3. Oblique or A-1

    4. A-3

    5. Palmar aponeurotic

     

    CorreCt answer: 3

     

    When the A-2 pulley remains intact, dividing either the A-1 or the oblique pulley will not alter thumb mechanical efficiency or joint angular displacement. If both the oblique pulley and A-1 pulley are cut, significant bow stringing will occur. Studies showed that repair or reconstruction of either the oblique pulley or the A-1 pulley after injury will restore thumb kinematics as long as the A-2 pulley is intact.

     

     

     

  31. Which of the following constitutes a positive intrinsic tightness test?

    1. Decreased proximal interphalangeal joint flexion with extension of the metacarpophalangeal joint

    2. Decreased proximal interphalangeal joint flexion with flexion of the metacarpophalangeal joint

    3. Normal proximal interphalangeal joint flexion with flexion of the metacarpophalangeal joint

    4. Normal proximal interphalangeal joint flexion with extension of the metacarpophalangeal joint

    5. Increased proximal interphalangeal joint flexion with extension of the metacarpophalangeal joint

    CorreCt answer: 1

     

    Extension of the metacarpophalangeal joint places tension on the intrinsic contribution to the extensor system via the lateral bands. Contracture of the intrinsics decreases the flexion at the proximal interphalangeal joint with the metacarpophalangeal joint placed in extension past neutral.

     

     

     

  32. Which of the following arteries is the pedicle supply to the lateral arm flap?

    1. Radial recurrent

    2. Profunda brachii

    3. Interosseous recurrent

    4. Anterior radial collateral

    5. Posterior radial collateral

    CORRECT ANSWER:

    5

    The posterior radial collateral artery provides the vascular supply to the lateral arm flap. The radial collateral artery travels with the radial nerve in the spiral groove until both penetrate the lateral intermuscular septum. It then divides into the anterior and posterior radial collateral arteries. The posterior branch passes posterior to the lateral intermuscular septum.

     

     

     

  33. A 34-year-old woman who underwent release of her first dorsal compartment at the wrist for de Quervain's tenosynovitis 3 months ago continues to report radial-sided wrist pain and tenderness similar to what she had prior to surgery. Examination appears classic for de Quervain's with a positive Finkelstein's test and continued pain with palpation over the first dorsal compartment. What is the likely source of her continued pain?

    1. Tendon subluxation

    2. Intersection syndrome

    3. Injury to the dorsal radial sensory nerve

    4. Tendon injury to the abductor pollicis longus (APL) tendon

    5. Unreleased extensor pollicis brevis (EPB) tendon

    CORRECT ANSWER: 5

    Persistant pain after first dorsal compartment release is often the result of failure to release all potential septations or compartments. It has been found that 24% to 34% of wrists have a separate compartment involving the EPB or APL. If each tendon is not identified, an incomplete release can result, causing continued symptoms. Intersection syndrome is more proximal, pain is not over the first dorsal compartment. Radial sensory nerve injury would not result in a positive Finkelstein's test. Tendon subluxation and tendon injury usually do not cause pain over the first dorsal compartment with palpation.

     

     

  34. Contracture or tightness of the triangular ligament of the finger is associated with which of the following conditions?

    1. Sagittal band insufficiency

    2. Volar subluxation of the lateral bands

    3. Swan-neck deformity

    4. Volar plate contracture

    5. Boutonniere deformity CORRECT ANSWER: 3

    Swan-neck deformity may result from contracture/tightness of the triangular ligament. Anatomically, the triangular ligament is on the dorsal aspect at the base of the middle phalanx just distal to the central slip. It keeps the lateral bands dorsal. With a boutonniere deformity, the lateral bands move volar to the central axis resulting in a flexion deformity of the proximal interphalangeal joint and extension of the distal interphalangeal joint. Sagittal band insufficiency results in subluxation of the extensor tendon(s) at the metacarpophalangeal joint level. Volar plate contracture will not cause swan-neck deformity.

     

     

     

  35. What is the most common site of posterior interosseous nerve compression in radial tunnel syndrome?

    1. Fibrous bands superficial to the radiocapitellar joint

    2. Radial recurrent artery branches (leash of Henry)

    3. Fibrous edge of the supinator (arcade of Frohse)

    4. Distal edge of the supinator

    5. Tendinous margin of the extensor carpi radialis brevis (ECRB)

    CORRECT ANSWER: 3

    The five compression sites described in radial tunnel syndrome are: the distal edge of the supinator; fibrous bands superficial to the radiocapitellar joint; tendinous margin of the extensor carpi radialis brevis (ECRB); radial recurrent artery (leash of Henry); and the most common site of compression, the fibrous edge of the supinator (arcade of Frohse). The tendinous portion of the supinator is next to the bone and does not compress the posterior interosseous nerve.

     

     

  36. What are the two most important pulleys to preserve/reconstruct during flexor tendon surgery?

    1. A1 and A2

    2. A2 and C3

    3. A2 and A3

    4. A2 and A4

    5. A3 and C4 CORRECT ANSWER: 4

    If the flexor tendon sheath is nonfunctional or a surgical approach to the flexor tendons is being performed, the minimum preservation/reconstruction should include the A2 and A4 pulleys. Ideally, the surgeon should reconstruct a pulley both proximal and distal to each joint to minimize bowstringing and maximize excursion.

     

     

     

  37. When releasing a proximal interphalangeal (PIP) joint flexion contracture, the check rein ligaments are released first, followed by which of the following structures?

    1. Proper collateral ligament from the proximal phalanx

    2. Proper collateral ligament from the middle phalanx

    3. Extensor tendon

    4. Dorsal capsule

    5. Accessory collateral ligament and volar plate

    CORRECT ANSWER: 5

    When releasing a PIP joint flexion constracture, each step should be followed by an attempt to extend the PIP joint. If there is no passive extension, then the next stage is performed. The steps for a volar approach PIP flexion contracture release are as follows: retract the flexor tendons after appropriate pulley takedown; release check rein ligaments; then accessory collateral ligament and volar plate; and finally the proper collateral ligament is then released off the proximal phalanx. Extensor tenolysis only needs to be performed if there is no active extension.

     

     

     

  38. A 25-year-old man has a painful mass in a web space of his foot. MRI scans are seen in Figures 19a (T2 STIR) and 19b (T1), a

    representative gross specimen is seen in Figure 19c, and a H&E stain is seen in Figure 19d. What is the most likely diagnosis?

     

     

     

     

     

     

     

     

     

     

     

     

    1. Melanoma

    2. Synovial sarcoma

    3. Interdigital neuroma

    4. Epithelioid sarcoma

    5. Giant cell tumor of tendon sheath

    CORRECT ANSWER: 5

    Giant cell tumors of tendon sheath are common in the hands and feet. Because of significant hemosiderin deposition, they commonly appear hypointense to skeletal muscle on both T1 and T2 pulse-weighted sequences. The hemosiderin is manifested in the brownish discoloration in the gross specimen. The photomicrograph shows bland spindled stromal cells and abundant multinuclear giant cells. Treatment is marginal excision with

    relatively low rates of tumor recurrence. Although the foot is not an infrequent site of melanoma and there are some shared radiologic features with giant cell tumor of tendon sheath, histologically melanoma is composed of cells both spindled and epithelioid arranged in nests or clusters. Synovial sarcoma is the most common sarcoma of the foot which radiographically has mineralizations in 30% of cases. It is typically heterogeneous on both MR pulse sequences.

    Microscopically, monophasic synovial sarcoma contains spindled cells that are arranged in short intersecting fascicles similar to fibrosarcoma.

    Pseudoglandular areas can be observed in biphasic cases. Epithelioid sarcoma, though common in the hand, is relatively rare in the foot and is histologically distinct from giant cell tumor of tendon sheath. When this tumor secondarily involves bone, it may be confused with osteomyelitis.

     

     

     

  39. Figures A through C show the MRI scans and biopsy specimen of a 9-year-old girl who has had progressive swelling and a mass on her medial elbow for 1 month. The area is increasingly painful to touch and with range of motion. The remainder of her examination is unremarkable. What is the next most appropriate step in management?

     

     

     

     

     

     

     

     

    1. Chemotherapy

    2. Radiation therapy

    3. Antibiotic therapy

    4. Observation

    5. Physical therapy CORRECT ANSWER: 3

    Cat scratch disease (CSD) is typically a benign and self-limited illness lasting 6 to 12 weeks in the absence of antibiotic therapy. Regional lymphadenopathy (axillary, epitrochlear, inguinal) is the predominant clinical feature of CSD; affected nodes are often tender and occasionally suppurate. Between 25% and 60% of patients report a primary cutaneous inoculation lesion (0.5- to 1-cm papule or pustule) at the site of a cat scratch or bite. The skin lesions typically develop 3 to 10 days after injury and precede the onset of lymphadenopathy by 1 to 2 weeks. Bartonella henselae is now regarded as the etiologic agent of CSD. For many years, CSD has been clinically diagnosed when three of the following four criteria are met in a patient: 1) history of traumatic cat contact;

    2) positive skin-test response to CSD skin-test antigen; 3) characteristic lymph node lesions; and 4) negative laboratory investigation for unexplained lymphadenopathy. Treatment consists of azithromycin, ciprofloxacin, doxycycline, or multiple other antibiotics, all of which have been used successfully. Radiation therapy and chemotherapy would be reserved for malignant diseases and would not be appropriate in this setting. Treatment is necessary for this infectious entity; therefore, observation or physical therapy is not indicated.

     

     

  40. A 45-year-old woman has a slowly enlarging mass over the radial aspect of her middle finger at the level of the proximal interphalangeal joint. It is associated with decreased flexion of the joint and clinically is fixed to the underlying bone. Radiographs reveal erosion of the lateral cortex of the proximal phalanx. Gross observation at the time of surgery reveals that the mass has a yellowish-brown tint and lobulated areas. Histology demonstrates bland fibrous stroma with scattered histiocytes, giant cells, and hemosiderin. What is the most likely diagnosis?

    1. Epithelioid sarcoma

    2. Giant cell tumor of tendon sheath

    3. Gouty tophus

    4. Hemangioma

    5. Epithelial inclusion cyst

    CORRECT ANSWER: 2

    The clinical and pathologic description is typical of a giant cell tumor of tendon sheath. Epithelioid sarcoma is the most common soft-tissue sarcoma in the hand and is composed of a nodular arrangement of tumor cells with epithelioid appearance and eosinophilia with a tendency to undergo central degeneration and ulceration. Gouty tophi have a characteristic white, chalky gross appearance and will demonstrate negatively birefringent crystals on polarized light microscopy. Hemangiomas are composed of a variable amount of fat and vessels. Epithelial inclusion cysts are filled with keratin from desquamation of the hyperkeratotic, stratified squamous epithelial cells that line the cysts.

     

     

     

  41. A 56-year-old right hand dominant male presents to your office complaining of right thumb pain worsened with pincer grip and using his mobile phone. He is a writer, and is having difficulty holding his pen. Radiographs from this visit are shown in Figure A. Compared with trapeziectomy alone, which of the following treatment options is likely to result in superior pain relief and improvement of key-pinch strength?

     

     

     

    1. Trapeziometacarpal corticosteroid injection followed by aggressive occupational therapy

    2. Trapeziectomy with interpositional palmaris longus arthroplasty

    3. Trapeziectomy, interpositional arthroplasty, and palmar oblique ligament reconstruction using flexor carpi radialis autograft

    4. Partial trapeziectomy with capsular interpositional arthroplasty

    5. None of the above CORRECT ANSWER: 5

    This patient has symptomatic basal joint arthritis with radiographic evidence of pantrapezial arthritis. Simple trapeziectomy has been shown to provide pain relief and improvement of key-pinch strength that is comparable to trapeziectomy plus interpositional arthroplasty.

     

    Definitive surgical management of basal joint arthritis commonly involves excision of the diseased trapezium with concomitant interpositional arthroplasty at the carpometacarpal joint in an effort to mantain the height of the metacarpal. This is commonly done with flexor carpi radialis (FCR) or palmaris longus (PL) autograft. Recent studies have called into question the need for interpositional arthroplasty, suggesting that excision of the trapezium alone can provide non-inferior results.

     

    Davis et. al. randomized 183 symptomatic trapeziometacarpal joints to one of three procedures: trapeziectomy alone, trapeziectomy with palmaris longus interpositional arthroplasty, or trapeziectomy with FCR interpositional arthroplasty and reconstruction of the palmar oblique ligament. For all patients, the thumb metacarpal was percutaneously pinned to the distal pole of the scaphoid to maintain the height of the digit. Patients were evaluated at three and 12 months post-operatively. At both time-points, they found no difference between groups with respect to subjective accounts of pain, function, stiffness, and weakness. Objective measures of thumb key-pinch strength were no different at either time point. The authors concluded that there may be no benefit to ligament reconstruction or tendon interposition in

    the short term.

     

    Li et. al. performed a systematic review of four randomized controlled trials and two systematic reviews to evaluate outcomes of trapeziectomy with and without LRTI for treatment of basal joint osteoarthritis. In their review, there were no statistically significant differences in post-op grip strength, pinch strength, visual analog pain scores, DASH scores, and complications. The authors concluded that both procedures produced similar clinical results.

     

    Raven et. al. performed a retrospective analysis of 54 patients who underwent one of three procedures for basal joint osteoarthritis: resection arthroplasty, trapeziectomy with tendon interposition, or trapeziometacarpal arthrodesis.

    The authors found resection arthroplasty to be a simple procedure with longterm results pain and functional outcomes comparable to trapeziectomy with tendon interposition.

     

    Naram et. al. retrospectively reviewed 200 patients who underwent simple trapeziectomy with or without LRTI and with or without Kirschner wire stabilization, or a Weilby ligament reconstruction. They found that patients undergoing trapeziectomy with LRTI or a Weilby procedure had a greater incidence of complications compared to trapeziectomy alone, including infection and reoperation.

     

    Figure A is a plain radiograph demonstrating pantrapezial arthritis with the thumb trapeziometacarpal joint being most significantly affected.

     

    Incorrect Answers:

    Answer 1: The patient has symptomatic debilitating basal joint arthritis with radiographic obliteration of the trapeziometacarpal joint, therefore cortisone injection would be unlikely to provide durable response.

    Answer 2 and 3: Trapeziectomy with interpositional arthroplasty using both PL or FCR are acceptable treatment options for trapeziometacarpal arthritis, however neither option is superior to trapeziectomy alone.

    Answer 4: Partial trapeziectomy with capsular interpositional arthroplasty has been proposed for treating basal joint arthritis, however the results of this technique have not been well studied and the indications are not well defined. In this patient, radiographs demonstrates moderate STT arthritis, suggesting that complete trapeziectomy would be a better solution than partial trapeziectomy at the trapeziometacarpal joint.

     

     

  42. A 31-year-old patient has had a left medial elbow mass for 1 month. The mass has been increasing in size and has now become very painful and erythematous. MRI scans are shown in Figures 76a and 76b. Laboratory studies show an erythrocyte sedimentation rate of 49 mm/h (normal 0 to 20 mm/h) and C-reactive protein level of 23 mg/L (normal 0 to 0.3 mg/L). Histology showed lymphoid tissue and multiple necrotizing granulomas. What organism is responsible for this clinical picture?

     

     

     

     

     

     

     

    1. Borrelia burgdorferi

    2. Trichophyton tonsurans

    3. Bartonella henselae

    4. Mycobacterium avium

    5. Corynebacterium minutissimum

    CorreCt answer: 3

     

    Cat scratch disease (CSD) is an important diagnosis for the orthopaedic surgeon to consider in the differential diagnosis of soft-tissue masses adjacent to epitrochlear or cervical lymph nodes. It is a soft-tissue tumor simulator and a high index of suspicion is necessary in all patients with upper extremity or head and neck adenopathy and a history of cat exposure. Although generally not required for diagnosis, cross-sectional imaging will reveal a mass with surrounding edema in an area of lymphatic drainage. A peripheral blood sample can be tested for Bartonella henselae - the offending organism with this diagnosis. Classically the histology of these lesions when biopsied will show multiple necrotizing granulomas. Mycobacterium avium is the only other organism that would demonstrate a granulomatous reaction and the location is classic for CSD. Borrelia burgdorferi is associated with Lyme disease.

    Mycobacterium avium may be a source of immunocompromised infections in HIV patients. Trichophyton tonsurans and corynebacterium minutissimum are not associated with orthopaedic diseases.

     

     

     

  43. A 45-year-old woman has a painful mass in the dorsum of the right wrist. It is firm and nontender to palpation. She states it has slowly gotten bigger over the past 3 years. You suspect a dorsal wrist ganglion. What is the most definitive way to confirm this diagnosis?

    1. Observe it for 1 year to see if it changes dramatically in size.

    2. Obtain a gadolinium enhanced MRI scan.

    3. Obtain radiographs, looking for scapholunate joint degenerative changes.

    4. Perform a needle aspiration and send the aspirate for cytologic examination.

    5. Apply direct firm manual pressure over the mass to see if it can be ruptured.

    CorreCt answer: 4

     

    Dorsal wrist ganglions are synovial cysts that arise most frequently from the scapholunate joint. They often extend between the extensor digitorum communis and extensor pollicis longus tendons at the wrist. Aspiration of the cyst is both oncologically safe if done appropriately and also the easiest way to definitively confirm the diagnosis. Clear, yellow viscous fluid/gel is most often aspirated. Cytologic evaluation is mandatory to exclude myxoid neoplasms.

    Because the lesion has been present for 3 years, further observation is not warranted. The classic presentation, physical examination findings, and location make MRI and radiographs unnecessary. Manual rupture of the mass is not recommended.

     

     

  44. A 28-year-old man fell while ice skating 6 months ago and has had ulnar-sided wrist pain ever since. The patient's wrist radiograph is shown in Figure A and a CT scan is shown in Figure B. What is the most appropriate treatment?

     

     

     

     

    1. Scapholunate ligament repair

    2. Excision of the hook hamate

    3. Excision of the pisiform

    4. Open reduction internal fixation of the hamate

    5. Open reduction internal fixation of the pisiform

    CORRECT ANSWER: 3

    Based on clinical history and imaging shown, this patient has developed a pisiform fracture nonunion. Treatment of symptomatic nonunions of the pisiform is by pisiformectomy

     

    Fractures of the pisiform are rare. They often occur in conjunction with injuries to the distal radius or carpus. Non-operative management with cast immobilization in 30 degrees of wrist flexion is the first line of treatment.

    Symptomatic nonunions are treated with pisiformectomy.

     

    Palmieri et al. performed pisiformectomies on 21 patients who had pisiform area pain that was refractory to conservative management. Patients had a history of painful union or nonunion of pisiform fractures, arthritis or FCU tendonitis. In all cases, wrist strength and mobility was retained.

     

    Lam et al. reviewed the effect of pisiform excision on wrist function in patients with piso-triquetral dysfunction. After an average follow up of 65 months, 75%

    of patients had complete relief of pisiform area symptoms. No differences in grip, wrist motion, strength or power were found in comparison to the contralateral side.

     

    Figure A shows an oblique radiograph of a pisiform fracture nonunion. Figure B shows an axial CT scan sequence of the wrist. A pisiform fracture nonunion is identified with subtle comminution. The pisotriquetral joint appears to be congruent.

     

    Incorrect Answers

    Answer 1: The scapholunate ligament is not affected in this clinical situation. Answers 2, 4: Although the hook of hamate can be a source of ulnar sided pain, it is not implicated in this clinical situation

    Answer 5: An ORIF of the pisiform is not typically used for symptomatic pisiform fracture nonunions

     

     

     

  45. A 32-year-old woman jammed her ring finger. Figures 77a and 77b show radiographs of the finger after a closed reduction. Which of the following interventions, if done correctly, is likely to result in the best possible final clinical outcome?

     

     

     

     

    1. Early removal of a splint and application of continuous passive motion

    2. Application of dynamic extension bracing after the first week

    3. Maintaining reduction of the middle phalanx on the condyles of the proximal phalanx with dynamic external fixation

    4. Open reduction and anatomic restoration of the middle phalanx articular surface

    5. Surgical advancement of the volar plate into the middle phalanx base

    CORRECT ANSWER: 3

    The most important determinant in the final clinical outcome in proximal interphalangeal (PIP) joint fracture locations is the maintenance of the PIP joint alignment on the lateral view. This can sometimes be done with just extension block splinting, sometimes the fracture requires dynamic external fixation, and sometimes the fracture requires open reduction or volar plate arthroplasty. Good function can be the result in the setting of an incongruent middle phalanx base as long as the PIP joint alignment is maintained.

    Continuous passive motion has not been shown to be of benefit. Whereas dynamic external fixation in a flexed position is a very good treatment, dynamic extension bracing will just precipitate loss of PIP joint reduction and is therefore not indicated. Whereas open reduction of the articular surface is theoretically desirable, it is generally impossible in the setting of the comminution of the volar middle phalanx base. Furthermore, open reduction and internal fixation by itself does not guarantee that the PIP joint alignment will be maintained, and typically it causes finger stiffness given the extensive surgical approach. Likewise, volar plate arthroplasty is a surgery of last resort and requires careful attention to PIP joint alignment before joint pinning. In this case, with characteristics of comminution, dynamic external fixation is the preferred choice.

     

     

     

  46. A 20-year-old woman sustained a laceration to her volar forearm 4 cm proximal to the wrist flexion crease. She has numbness in the thumb, index, and middle fingers. After microscopic repair of the median nerve, 2 weeks of splinting, and commencement of a hand therapy program, the patient is most likely to require what secondary operation 6 months after the injury?

    1. Tenolysis of the profundus tendons at the wrist

    2. Nerve transfer of the ulnar motor nerve to the median motor nerve

    3. Opponensplasty with the extensor indicis

    4. Open carpal tunnel release

    5. Transfer of the extensor digiti minimi to the first dorsal interosseous tendon

    CORRECT ANSWER: 3

    The patient sustained a laceration of the median nerve in what would be considered a low median nerve injury. Standard treatment entails exploration and microscopic repair of the median nerve. With a good quality nerve repair in a young adult, return of some sensory function (albeit reduced compared with the normal nerve) is usual. Return of motor function to the thenar muscles is more unpredictable. If the patient begins a therapy program within a few weeks after nerve repair, it is unlikely that tenolysis of the profundus tendons would be required. An open carpal tunnel release would be unlikely to change functional return. The patient would not be expected to have lost first dorsal interosseous function after a median nerve laceration because this muscle is innervated by the ulnar nerve. A neurotization procedure for low median nerve palsy has been described, but it consists of transfer of the distal anterior interosseous nerve into the median nerve motor fascicles, not transfer of the ulnar nerve. Therefore, the most likely secondary procedure required in this scenario is an opponensplasty procedure to improve thumb opposition.

     

     

     

  47. What is the most efficient pressure for use with negative pressure wound therapy?

    1. 25 mm Hg

    2. 75 mm Hg

    3. 125 mm Hg

    4. 300 mm Hg

    5. 500 mm Hg CORRECT ANSWER: 3

    In animal and clinical studies, a range of pressures between 50 mm Hg to 500 mm Hg were tested; the most efficient pressure was 125 mm Hg, resulting in a fourfold increase in blood flow, 63% increase in granulation tissue with continuous pressure, and 103% increase in granulation tissue with intermittent pressure. When 125 mm Hg pressures were compared with either those less than 50, or those greater than 250, there was a decrease in granulation tissue in swine models.

     

     

     

  48. Figures 125a and 125b are the current radiographs of a 52-year-old man who sustained an injury to his dominant wrist 8 weeks ago. He is an alcoholic and does not remember the details of how he injured it. Paperwork showing what treatment he received at an

    urgent care facility indicates that he was given a splint for his "sprained wrist." Examination reveals the pain is getting better, but there is persistent swelling and range of motion is very limited.

    Recommended treatment at this time should consist of

     

     

     

     

     

     

     

    1. discontinuation of the splint and commencement of a regimen of hand therapy.

    2. casting for an additional 2 weeks and reassessment of the fracture healing at that time.

    3. open reduction and internal fixation of the injury.

    4. proximal row carpectomy.

    5. wrist arthrodesis.

     

    CorreCt answer: 4

    The injury represents a very uncommon presentation of a perilunate injury pattern. Whereas these injuries are sometimes overlooked on initial radiographic studies, they are usually recognized much sooner. In this case of a late presenting perilunate injury in a patient that is not entirely responsible, a proximal row carpectomy represents the best treatment option. Open reduction and internal fixation is generally not successful because of cartilage degeneration and contracture that has developed in the interim. No further splinting or casting is indicated, and neglecting the injury would be indicated only if the patient refused any further treatment. Wrist arthrodesis is generally indicated only as a salvage procedure if a proximal row carpectomy is unsuccessful.

     

     

     

  49. A 47-year-old woman sustained a nondisplaced distal radius fracture 6 months ago and is unable to extend her thumb. When performing reconstruction using the extensor indicis proprius to the extensor pollicis longus transfer, tension is ideally determined by securing the tendons in what manner?

    1. In maximum tension with the wrist and thumb in extension

    2. In maximum tension with the wrist and thumb in neutral

    3. In maximum tension with the wrist and thumb in flexion

    4. According to the tenodesis effect with wrist flexion and extension

    5. According to functional testing with the patient awake under local anesthesia

    CorreCt answer: 5

     

    Extensor pollicis longus rupture can result from distal radius fractures. Synergistic tendon transfer can be achieved using the extensor pollicis longus as the motor donor. Whereas different schemes for achieving optimal tension are available, the most reliable method is to tension the repair under local anesthesia while asking the patient to perform thumb flexion and extension. Tendon transfer tension can be adjusted accordingly to achieve maximum extension without compromising active flexion range. Other methods of tensioning are estimates at best, and maximum tensioning in patients without neuromuscular disease is rarely used in tendon transfers.

     

     

     

  50. Which of the following substances is likely to cause the most soft-tissue damage in the long term if injected into a fingertip under

    high pressure?

     

    1. Grease

    2. Latex paint

    3. Water

    4. Oil-based paint

    5. Chlorofluorocarbon-based refrigerant

    CORRECT ANSWER: 4

    This type of injury represents a difficult problem in hand surgery. The factors that most determine outcome after high-pressure injection injuries into the fingertip include: involvement of the tendon sheath, extent of proximal spread of the injected substance, pressure setting, and delay to surgical treatment.

    The other factor that likely is most important is the type of substance injected. Water and latex-based paints are least destructive. Grease and chlorofluorocarbon-based substances are intermediate, but aggressive surgical debridement can restore reasonable function. Oil-based paints are highly inflammatory and can cause such chronic inflammation such that amputation may be the only reasonable treatment option despite early aggressive surgical treatment.

     

     

     

  51. A 37-year-old woman has right-hand numbness and tingling. Based on the history and examination, carpal tunnel syndrome is suspected, and electrodiagnostic tests also point to the same diagnosis. The patient has worn night splints for the last 8 weeks with continued persistent symptoms. What is the next most appropriate step in management?

    1. Continue the night splinting for 1 additional month.

    2. Continue the night splinting for 3 more months.

    3. Switch to full-time splinting and reevaluate in 1 month.

    4. Switch to full-time splinting for 3 more months.

    5. Perform carpal tunnel release.

     

    CorreCt answer: 5

     

    Various nonsurgical management options exist for carpal tunnel syndrome (local and oral steroids, splinting, and ultrasound). All effective or potentially effective nonsurgical forms of management have measureable effects on symptoms within 2 to 7 weeks of the initiation of treatment. If a treatment is not effective within that time frame, a different treatment option should be

    chosen. In this case, continued splinting is unlikely to improve symptoms and steroid injection or surgery is indicated.

     

     

     

  52. A 46-year-old man sustains an injury to his left index finger while cleaning his paint gun with paint thinner. Examination reveals a small puncture wound at the pulp. The finger is swollen. What is the next most appropriate step in management?

    1. Elevation and observation

    2. Surgical debridement and lavage

    3. Infiltration with corticosteroids

    4. Infiltration with a neutralizing agent

    5. Administration of antibiotics

    CORRECT ANSWER: 2

    High-pressure injection injuries are associated with a high risk of amputation. The risk of amputation is highest with organic solvents. The presence of infection and the use of steroids do not impact the amputation rate.

    Amputation risk is lower if surgical debridement is performed within 6 hours. Elevation and observation would delay necessary care. Neutralizing agents may be used in specific situations, such as hydrofluoric acid exposure or chemotherapeutic agent extravasation, but in high pressure paint thinner injection, the best outcome is achieved through early surgical lavage.

     

     

     

  53. A 54-year-old woman who has a history of undergoing left trapezium excision with ligament reconstruction and tendon interposition using the entire flexor carpi radialis performed by another surgeon, now reports left basilar thumb pain. Examination reveals pain and subluxation of the carpometacarpal joint with axial loading. The metacarpophalangeal joint hyperextends to 60 degrees, but radiographs show intact joint space. What is the best option to improve function?

    1. Bracing with a hand-based thumb spica splint

    2. Pinning of the carpometacarpal joint

    3. Pinning of the carpometacarpal and metacarpophalangeal joints

    4. Carpometacarpal revision stabilization

    5. Carpometacarpal revision stabilization and metacarpophalangeal joint fusion

    CorreCt answer: 5

     

    The patient previously underwent ligament reconstruction and tendon interposition. However, the previous surgeon failed to address metacarpophalangeal joint hyperextension, which leads to adduction contracture and collapse of the basilar joint. With the basilar joint causing pain and instability, repeat ligament reconstruction should be performed. Splinting alone is unlikely to resolve instability problems. Because the flexor carpi radialis was used, the next option is to use the abductor pollicis longus.

    Additionally, the severe metacarpophalangeal joint hyperextension should be corrected by fusion. Simple pinning is unlikely to provide long-term stability when this degree of hyperextension exists.

     

     

     

  54. When evaluating a patient with suspected purulent flexor tenosynovitis in the thumb, the distal forearm and little finger are found to be swollen as well. The most likely anatomic explanation is the existence of a potential space in which of the following?

    1. Through the carpal tunnel

    2. Across the midpalmar space

    3. Communicating with the subcutaneous tissue

    4. Superficial to the distal antebrachial fascia

    5. Between the fascia of the pronator quadratus and flexor digitorum profundus conjoined tendon sheaths

    CorreCt answer: 5

     

    Pyogenic flexor tenosynovitis is an infection within the flexor tendon sheath that can involve the fingers or thumb. The tendon sheaths begin at the metacarpal neck level and extend to the distal interphalangeal joint. In the little finger and the thumb, the sheaths usually communicate with the ulnar and radial bursae, respectively. The potential space of communication, Parona's space, lies between the fascia of the pronator quadratus muscle and flexor digitorum profundus conjoined tendon sheaths. Infection tracking through this space presents as a horseshoe abscess.

     

     

     

  55. Which of the following proximal phalanx fractures can most reliably be treated with a closed reduction and avoidance of surgical measures?

    1. Midshaft transverse diaphyseal fracture with 30 degrees of angulation

    2. Long spiral diaphyseal fracture with 15 degrees of malrotation

    3. Open fracture with skin loss and exposed extensor tendon

    4. Distal condylar intra-articular fracture with minimal displacement

    5. Proximal metaphyseal fracture location with 30 degrees of dorsal tilting

    CORRECT ANSWER: 1

    Proximal phalanx fractures are very common, but care must be taken to understand which injuries are reliably treated with nonsurgical measures, and which ones are prone to clinically symptomatic malunion without surgical treatment. The proximal metaphyseal location is a problematic fracture to get reduced with closed measures, and due to the forces of the extensor apparatus, is prone to collapse into the original deformity. Imaging is also frequently difficult because of the overlap of the other fingers and frequently the true angulation is underappreciated. With 30 degrees of angulation, consideration should be given to surgical treatment. Long oblique/spiral fractures with malrotation are also most reliably treated with multiple lag screws, because maintaining the reduction with nonsurgical measures is unreliable, and can lead to significant functional problems in the form of crossover of the fingers with gripping. Open fractures with skin loss clearly are treated with surgical measures. Distal condylar fractures with minimal displacement are another fracture pattern that have a high rate of loss of reduction when treated nonsurgically. Like most articular fractures, they are best treated with anatomic reduction and rigid internal fixation. By comparison, closed midshaft transverse diaphyseal fractures can usually be anatomically reduced and held in this position with closed measures.

     

     

     

  56. Figure 3 shows an arthroscopic view of the radiocarpal joint from the 3-4 portal, looking volarly and radially (Sc=scaphoid, R=Radius). What structure is marked by the asterisk?

     

     

     

    1. Radioscaphocapitate ligament

    2. Scapholunate ligament

    3. Palmar oblique ligament

    4. Dorsal intercarpal ligament

    5. Triangular fibrocartilage complex (TFCC)

    CORRECT ANSWER: 1

    The radioscaphocapitate ligament is a volar capsular structure running obliquely from the radial styloid to the scaphoid waist, ultimately inserting on the proximal radial aspect of the capitate. The radioscaphocapitate ligament is important in preventing ulnar translocation of the carpus. The scapholunate ligament is located intra-articularly, between the scaphoid and lunate. The dorsal intercarpal ligament is a dorsal structure, and not visible during routine wrist arthroscopy. The palmar oblique ligament connects the first and second metacarpal bases. The TFCC is visible during wrist arthroscopy between the radius and ulna.

     

     

     

  57. A 22-year-old man reports a 2-week history of a burning pain along the dorsoradial aspect of the distal forearm. The pain radiates to the dorsum of the thumb. Examination reveals tenderness and reproduction of symptoms with percussion 8 cm proximal to the radial styloid. Reproduction of symptoms also occurs with forearm pronation

    and ulnar deviation of the wrist. No discrete sensory deficit is noted and electrodiagnostic studies are normal. Nonsurgical management consisting of rest, splinting, and anti-inflammatory medications for 6 weeks has failed to provide relief. Treatment should now consist of decompression of the

    1. lateral antebrachial cutaneous nerve in the interval between the abductor pollicis longus and the extensor pollicis brevis in the forearm.

    2. lateral antebrachial cutaneous nerve in the interval between the brachioradialis and the extensor carpi radialis longus in the distal forearm.

    3. radial sensory nerve in the interval between the extensor carpi radialis longus and the extensor carpi radialis brevis in the distal forearm.

    4. radial sensory nerve in the interval between the brachioradialis and the extensor carpi radialis longus in the distal forearm.

    5. radial sensory nerve in the interval between the brachioradialis and the extensor carpi radialis brevis in the distal forearm.

    CorreCt answer: 4

     

    Wartenberg's syndrome, or compression of the sensory branch of the radial nerve, occurs in the interval between the brachioradialis and the extensor carpi radialis longus approximately 8 cm proximal to the radial styloid. There may be history of repetitive wrist/forearm circumduction activity (ie, knitting) or of wearing a tight wristwatch or jewelry. It can occur in patients who have been handcuffed. Typical clinical findings are pain, paresthesia, and/or hypesthesia in the dorsoradial aspect of the wrist and hand in the distribution of the radial sensory nerve. There is often a positive Tinel's sign over the compression site. Hypesthesia may be present in the distribution of the radial sensory nerve which is typically on the dorsal aspect of the first dorsal web space and dorsum of the thumb; however, with overlap in the distribution of the superficial radial nerve and the lateral cutaneous nerve of the forearm this may not always be present. Surgical management consists of release of the nerve as it exits the interval between the brachioradialis and the extensor carpi radialis longus in the distal forearm.

     

     

     

  58. A 55-year-old woman with rheumatoid arthritis reports that she awoke with an inability to flex the interphalangeal joint of her thumb. Figure 8 shows an intraoperative finding. What is the most appropriate surgical treatment?

     

     

     

    1. Primary repair of the tendon

    2. Tendon reconstruction with the palmaris longus tendon

    3. Tendon reconstruction using a transfer of the flexor digitorum profundus (FDP) of the ring finger

    4. Thumb metacarpophalangeal fusion

    5. End-to-side repair of the flexor pollicis longus to the FDP of the index finger

    CORRECT ANSWER: 2

    The patient has sustained a chronic flexor pollicis longus rupture (Mannerfelt lesion). The injury is most likely a result of tendinopathy and attritional rupture of the tendon secondary to synovitis and bony osteophytosis at the scaphotrapeziotrapezoid joint. Because of the attritional injury and inherent tendinopathy, primary repair is unlikely to be successful. Among the options listed, tendon graft reconstruction with the palmaris longus tendon is the most appropriate treatment. Tendon reconstruction is possible with the flexor digitorum profundus of the index finger, not the flexor digitorum profundus of the ring finger. If osteophytes are encountered, these should be debrided.

    Thumb interphalangeal fusion is an option, but metacarpophalangeal fusion is not beneficial. End-to-side repair of the flexor pollicis longus to the FDP of the index finger is not appropriate and would sacrifice needed function of the index finger.

     

     

     

  59. Figures A and B show the initial radiographs of a 27-year-old snow boarder who fell backward onto his left outstretched hand. Which of the following most accurately describes the sequence of events that occurred during this injury?

     

     

     

     

     

     

    1. Lunotriquetral ligament failure followed by distal row dissociation, scaphoid extension, scaphoid failure, and dorsal dislocation of the carpus

    2. Volar dislocation of the lunate followed by scaphoid extension, scaphoid failure, lunotriquetral failure, and distal row dissociation

    3. Dorsal intercarpal ligament failure followed by distal row dissociation, scaphoid failure, lunotriquetral ligament failure, and dorsal dislocation of the carpus

    4. Short radiolunate ligament failure followed by volar dislocation of the lunate, lunotriquetral ligament failure, scaphoid failure, and distal row dissociation

    5. Scaphoid extension followed by scaphoid failure, distal row dissociation, lunotriquetral ligament failure, and dorsal dislocation of the carpus

    CorreCt answer: 5

     

    As described by Mayfield and associates, the typical sequence of events referred to as "progressive perilunar instability" that result in a volar

    perilunate dislocation are as follows: scaphoid extension, followed by opening of the space of Poirer, scaphoid failure, and distal row dissociation, which in turn lead to hyperextension of the triquetrum, lunotriquetral ligament failure, and finally dorsal dislocation of the carpus. The lunate remains in the lunate fossa in a perilunate fracture-dislocation but is dislocated in a lunate dislocation. The short radiolunate and dorsal intercarpal ligaments typically remain intact.

     

     

     

  60. Which of the following is the most consistently proposed tendon transfer for radial nerve palsy?

    1. Pronator teres to extensor carpi radialis brevis

    2. Brachioradialis to extensor carpi radialis brevis

    3. Flexor carpi radialis to extensor digitorum communis

    4. Palmaris longus to extensor pollicis longus

    5. Flexor digitorum superficialis to abductor pollicis longus and extensor pollicis brevis

    CorreCt answer: 1

     

    Whereas there are many variations of tendon transfers for radial nerve palsy, the most consistently proposed tendon transfer is the pronator teres to extensor carpi radialis brevis. The brachioradialis is innervated by the radial nerve so that is not an option. The flexor digitorum superficialis, flexor carpi radialis, and flexor carpi ulnaris are appropriate options to transfer to the extensor digitorum communis. The palmaris longus is not always present. A transfer to the abductor pollicis longus and extensor pollicis brevis may not be necessary if the extensor pollicis longus is rerouted to allow for abduction of the first ray.

     

     

     

  61. A patient has severe cubital tunnel syndrome and marked wasting of the intrinsic muscles of the hand. Why is the little finger held in an abducted position?

    1. Accessory slip of the extensor digiti minimi attaching to the abductor digiti minimi tendon

    2. Tetanic contraction of the abductor digiti minimi

    3. Radial collateral ligament insufficiency of the fifth metacarpophalangeal (MCP) joint

    4. Unopposed pull of the flexor digitorum profundus

    5. Muscle innervation from a Martin-Gruber anastomosis

    CORRECT ANSWER: 1

    A Wartenberg's sign, where the little finger is held in an abducted position, is associated with an ulnar nerve palsy. This happens when there is an accessory slip of the extensor digiti minimi, which is innervated by the radial nerve, crossing ulnar to the center of the MCP joint to attach to the tendon of the abductor digiti minimi and the proximal phalanx. The abductor digiti minimi and the volar interosseous muscles are both innervated by the ulnar nerve; therefore, there is no tetanic contraction of the abductor digiti minimi.

    Unopposed pull of the flexor digitorum profundus results in excess flexion of the proximal interphalangeal and distal interphalangeal joints of the hand as seen with a clawing-type deformity. A Martin-Gruber anastomosis, which is a neural connection between the ulnar and median nerves in the forearm, cannot explain this finger position.

     

     

     

  62. Figure 38 shows the radiograph of a 41-year-old man who reports ulnar palmar pain, decreased sensibility and tingling in the ring and little fingers, and a grating sensation in the ulnar fingers with motion. He reports that he sustained a fall on an outstretched hand 6 months ago. What is the most appropriate treatment option?

     

     

     

     

    1. Ulnar gutter cast

    2. Short arm cast

    3. Carpal tunnel release

    4. Decompression of Guyon's canal

    5. Excision of a fractured hook of hamate

    CORRECT ANSWER: 5

    Excision of a fractured hook of hamate is the most appropriate management. The patient has a hook of hamate fracture with ulnar nerve compression and irritation of the flexor tendons by the fracture surfaces; this puts the tendons at risk for rupture. Cast treatment will most likely not gain union of the fracture and will not address the nerve or tendon problems. Decompression of Guyon's canal alone will not address the tendon issue.

     

     

     

  63. A 25-year-old man was involved in an altercation. Examination reveals loss of active extension of the middle finger metacarpophalangeal (MCP) joint. A diagnosis of sagittal band rupture is made. Which of the following is considered the key diagnostic finding?

    1. Extensor lag of 30 degrees

    2. Extensor lag of 60 degrees

    3. Positive Bunnell intrinsic tightness test

    4. Ability to maintain active extension of the interphalangeal joints

    5. Ability to maintain MCP extension after passive extension

    CORRECT ANSWER: 5

    In sagittal band rupture, the extensor tendon may subluxate into the valley between the metacarpal heads. The patient will not be able to actively extend the MCP joint from a flexed position with the subluxated tendon, but will be able to maintain MCP extension after it has been passively extended. Extensor lags can have other etiologies other than extensor digitorum communis subluxation such as tendon laceration or rupture, posterior interosseous nerve palsy, but in these conditions, patients cannot maintain MCP extension. Active interphalangeal extension can be achieved with the intrinsic muscles that are not affected by sagittal band rupture.

     

     

     

  64. What is the effect of shortening of metacarpal fractures?

     

    1. Causes the greatest degree of extensor lag in the index finger

    2. Causes the greatest degree of extensor lag in the little finger

    3. Results in an average extensor lag of 7 degrees for every 2 mm of shortening

    4. Results in an average extensor lag of 14 degrees for every 2 mm of

      shortening

    5. Has no effect on grip strength

    CORRECT ANSWER: 3

    Cadaveric models have demonstrated a 7-degree extensor lag for every 2 mm of metacarpal shortening, with the amount of lag increasing in a linear fashion. There was no statistical difference in the amount of lag in regard to the digit involved. Based on muscle length-tension relationships, cadaveric models have also been used to demonstrate an 8% loss of power secondary to decreased interosseous force generation with 2 mm of shortening. Because the intrinsic muscles of the hand contribute anywhere from 40% to 90% of grip strength, decreased interosseous force generation secondary to metacarpal shortening will invariably cause a decrease in grip strength.

     

     

     

  65. A 22-year-old motorcyclist sustains open fractures to the left radial shaft and second and third metacarpals with exposed extensor tendon and bone. The fractures are approached via the dorsal open wounds of the forearm and hand with no additional incisions made. The radiograph and clinical photograph of the remaining defect in the hand are shown in Figures 55a and 55b. The remaining wound can be most appropriately covered with which of the following?

     

     

     

     

     

     

    1. Split-thickness skin grafting

    2. Posterior interosseous rotational flap

    3. Radial forearm rotational flap

    4. Groin flap

    5. Free lateral arm flap CORRECT ANSWER: 3

    After adequate debridement, there is exposed bone, tendon, and hardware. Split-thickness skin grafting over exposed tendon will not have a viable bed to support the graft. The tendons would not have healthy surrounding tissue, resulting in poor tendon gliding. The dorsal wound has disrupted the posterior interosseous artery that runs in the septum between the extensor digiti minimi and the extensor carpi ulnaris. Following the reconstructive ladder, the radial forearm rotational flap accomplishes wound coverage with a local flap rather

    than a groin flap (a distant flap) or a lateral arm flap (microvascular free tissue transfer).

     

     

     

  66. What is the effect of performing a flexor tenosynovectomy with an open carpal tunnel release for idiopathic carpal tunnel syndrome?

    1. Increased risk of nerve injury

    2. Improved postoperative finger flexion

    3. No added long-term clinical benefit versus open carpal tunnel release alone

    4. Increased postoperative pain

    5. Decreased recurrence of carpal tunnel syndrome

    CORRECT ANSWER: 3

    In patients with idiopathic carpal tunnel syndrome, flexor tenosynovectomy has not been shown to change the clinical outcome compared with open carpal tunnel release alone. This has been demonstrated in a randomized clinical trial of open carpal tunnel release with or without flexor tenosynovectomy. There has also been no evidence to suggest there is an added risk to performing the flexor tenosynovectomy. At time of surgery, the gross or histologic appearance of the flexor tenosynovium does not correlate with preoperative symptoms nor with clinical outcomes. The histology of the tenosynovium has been shown to be that of fibrosis in a setting of chronic inflammatory changes and no evidence of an acute inflammatory process exists. There may be an added role for flexor tenosynovectomy in non-idiopathic carpal tunnel syndrome such as in patients with renal disease or diabetes.

     

     

     

  67. Figures 69a and 69b show the radiographs of a 62-year-old man with severe radially sided wrist pain. Management has consisted of wrist splinting, nonsteroidal anti-inflammatory drugs, and activity modification, but he continues to have pain and reports difficulty sleeping. What is the most appropriate treatment for this patient?

     

     

     

    1. Arthroscopic debridement

    2. Open reduction and internal fixation

    3. Scaphoid nonvascularized bone graft and screw fixation

    4. Scaphoid vascularized bone graft and screw fixation

    5. Scaphoid excision and 4-corner fusion

    CORRECT ANSWER: 5

    Scaphoidectomy and 4-bone fusion is the most appropriate management based on the choices available. The patient has arthritic changes of SNAC (scaphoid nonunion advanced collapse) wrist, stage III. Stage I is at the radial styloid, stage II is at the radioscaphoid joint, and stage III is at the midcarpal joint. Arthroscopic debridement is not appropriate in patients with arthrosis.

    Attempting to achieve scaphoid union is only appropriate if there is no arthrosis or the changes are classified as stage I where radial styloidectomy can be performed.

     

     

     

  68. A 7-year-old boy is referred to your office 3 months after jamming his finger while playing basketball. Examination reveals 40 degrees of active and passive motion at the proximal interphalangeal (PIP) joint. The PIP joint is stable to radial and ulnar stressing. Radiographs are shown in Figures 76a and 76b. What is the most appropriate management?

     

     

     

     

     

     

    1. Observation

    2. Corrective osteotomy

    3. Ostectomy

    4. Hand therapy for aggressive stretching

    5. Dynamic splinting CORRECT ANSWER: 3

    The most appropriate management is an ostectomy, or resection of the bone in the subcondylar fossa region. This is a malunion where the subcondylar fossa is blocked by malaligned bone. Because it is a bony block to motion, stretching or dynamic splinting will be of no benefit. The physis of the proximal phalanx is proximal, making remodeling of a fracture at the distal end very

    unlikely. A corrective osteotomy has a risk of osteonecrosis of the very small distal fragment.

     

     

     

  69. Figure 78 shows the clinical photograph of a patient who injured his finger while playing football. He cannot actively flex the distal interphalangeal joint of the ring finger. Which of the following is the most accurate statement regarding the injury shown?

     

     

     

     

    1. The tendon is attached to the avulsed fragment from the distal phalanx.

    2. There is no difference in time sensitivity in an acute injury whether or not the tendon has retracted into the palm.

    3. In a chronic (> 3 months) case of flexor digitorum profundus (FDP) avulsion, the FDP should be tenodesed to the flexor digitorum sublimis (FDS).

    4. If the FDP is advanced more than 1.5 cm, there is a risk for quadriga effect.

    5. The method of repair does not affect repair gapping or strength of the tendon repair.

    CorreCt answer: 4

     

    Overadvancement of the FDP tendon is one of the causes of the quadriga effect. Relative shortening of an FDP tendon decreases the excursion of the neighboring FDP tendons because they originate from a common muscle belly. The patient reports a weak grasp. Answer 1 is not correct because there can be a fracture and the tendon can avulse off of the fracture fragment (Trumble JHS-A 1992). Whether the tendon has retracted into the palm or not does matter because retraction into the palm allows pulleys to collapse and contract and it also means that the vinculae have been stripped off of the tendon.

    Regarding answer 3, in chronic cases where the FDS is intact and strong, many patients may be better off with a sublimis finger and no FDP reconstruction that could, in the worst case scenario, worsen a functional proximal interphalangeal joint. Regarding the repair method, there is recent

    research showing method of repair (button vs anchor), suture type, and method do affect the biomechanical properties of the repair.

     

     

     

  70. A 44-year-old woman with cubital tunnel syndrome and associated ulnar nerve subluxation with elbow flexion has failed to respond to nonsurgical management. Which of the following statements is most acccurate regarding in situ simple decompression of the nerve compared with subcutaneous anterior transposition?

    1. Patients undergoing anterior transposition have improved motor outcomes.

    2. Patients undergoing anterior transposition have improved sensory outcomes

    3. Patients undergoing simple decompression have improved motor outcomes.

    4. Patients undergoing simple decompression have improved sensory outcomes.

    5. No differences in outcome are likely between treatment types.

     

    CorreCt answer: 5

     

    Recent reports comparing outcomes of surgical treatment of ulnar nerve compression at the elbow have demonstrated no differences in outcome between simple decompression and anterior transposition. The presence of subluxation of the ulnar nerve was not a contraindication to in situ decompression in the study by Keiner and associates.

     

     

     

  71. What anatomic structure must be excised when performing a volar plate arthroplasty of the proximal interphalangeal joint?

    1. Central slip

    2. Collateral ligament

    3. Checkrein ligament

    4. Triangular ligament

    5. Flexor digitorum superficialis insertion

    CORRECT ANSWER: 2

    The collateral ligament must be excised or released from the proximal phalanx to allow gliding of the middle phalanx on the articular surface of the proximal phalanx. Failure to do so may prevent this gliding motion and make the middle phalanx just hinge on the proximal phalanx.

     

     

  72. Figures 97a and 97b show a clinical photograph and radiograph of a patient who has a history of repeated drainage from the lesion. What is the preferred surgical treatment?

     

     

     

     

     

     

     

    1. Excision of the lesion alone

    2. Removal of the osteophyte alone

    3. Distal interphalangeal joint fusion

    4. Excision of the mass and osteophyte removal

    5. Removal of the mass and skin with skin grafting

    CORRECT ANSWER: 4

    The patient has a mucoid cyst. Whereas many of these lesions are associated with osteoarthritis, the best surgical treatment of the lesions in patients who have little or no pain is typically excision of the mass with osteophyte removal. Studies have shown that osteophyte excision helps minimize the risk of recurrence. Distal interphalangeal joint fusion is reserved for patients with pain and more advanced radiographic arthritis. Excision of the lesion alone is a less favorable option than excision of the mass and osteophyte removal. The lesion is independent of the skin and thus, skin removal with the mass is unnecessary.

     

     

     

  73. Which of the following structures cannot be seen during standard radiocarpal arthroscopy?

    1. Scapholunate ligament

    2. Lunotriquetral ligament

    3. Radioscaphocapitate ligament

    4. Extensor carpi ulnaris tendon

    5. Superficial insertion of the triangular fibrocartilage complex (TFCC)

    CORRECT ANSWER: 4

    The extensor carpi ulnaris tendon is located in an extra-articular position, and as such, cannot be seen during arthroscopy. Wrist arthroscopy is a useful technique for evaluation and treatment of radiocarpal and midcarpal maladies. During standard radiocarpal arthroscopy, the scapholunate and lunotriquetral ligaments can be easily visualized. The superficial TFCC is seen overlying the ulnar head. Volarly, the radioscaphocapitate ligament can be seen as a discrete band of the capsule.

     

     

     

  74. A 20-year-old skateboarder fell 6 months ago and has had radial-sided wrist pain since. His radiograph upon presentation to your office is shown in figure A. What is the most appropriate treatment at this time?

     

     

     

    1. four corner fusion

    2. long arm thumb spica cast

    3. wrist arthroscopy to evaluate intercarpal ligaments

    4. open reduction internal fixation with autologous bone graft

    5. wrist arthrodesis CORRECT ANSWER: 4

    This patient has a scaphoid waist fracture nonunion. Several studies indicate that scaphoid nonunions left untreated have a determined course of collapse and progressive arthritis (scaphoid nonunion advanced collapse - SNAC). Per Markiewitz et al, the standard treatment of scaphoid nonunions is open reduction internal fixation with bone graft; non-operative treatment is not appropriate. Proximal row carpectomy and wrist fusion are salvage procedures reserved for patient that has an advanced scaphoid nonunion, collapse and wrist arthritis.

     

     

     

  75. Figures 112a and 112b show the radiographs of a 28-year-old motorcyclist who sustained a closed hand injury in a collision. What is

    the most appropriate definitive treatment?

     

     

     

     

     

     

     

    1. Closed reduction and a hand/forearm cast in the intrinsic plus position

    2. Closed reduction and a hand splint

    3. Primary fusion of the carpometacarpal joints

    4. Closed versus open reduction and internal fixation

    5. Closed reduction and external fixation

    CORRECT ANSWER: 4

    Closed versus open reduction and internal fixation is the most appropriate treatment. The radiographs show fracture-dislocations of all five carpometacarpal joints. These injuries are extremely unstable and not amenable to closed (splint or cast) treatment only. External fixation may be warranted in an open, contaminated injury. Fusion would be an option if this were a chronic, painful condition on presentation.

     

     

     

  76. What additional procedure should be done when performing a radioscapholunate fusion for posttraumatic arthrosis following a distal radius fracture?

    1. Excision of the triquetrum and distal pole of the scaphoid

    2. Anterior interosseous neurectomy

    3. Fascial interposition arthroplasty of the capitolunate joint

    4. Sectioning of the dorsal intercarpal ligament

    5. Ulnar shortening osteotomy

    CORRECT ANSWER: 1

    Excision of the triquetrum and distal pole of the scaphoid frees up the midcarpal joint, improving radial deviation and the flexion-extension arc of motion of the wrist. This offers an alternative to complete wrist arthrodesis for posttraumatic arthrosis of the radiocarpal joint. An anterior interosseous neurectomy is believed to decrease some pain transmission from the wrist but because the fusion is done dorsal, cutting this volar structure is not routinely done. Fascial interposition is not needed because the capitolunate should be preserved in posttraumatic radiocarpal arthrosis. Sectioning of the dorsal intercarpal ligament would provide no benefit. If the triquetrum is excised, then an ulnar shortening osteotomy is unnecessary.

     

     

     

  77. Apert's syndrome is caused by a mutation in what gene?

     

    1. Fibroblast growth factor receptor 2 (FGFR2)

    2. Fibroblast growth factor receptor 3 (FGFR3)

    3. Collagen type II alpha 1 chain (COL2A1)

    4. SED late (SEDL)

    5. Fibrillin

     

    CorreCt answer: 1

     

    Apert's syndrome (acrocephalosyndactyly type 1) is characterized by anomalies of the cranium, hands, and feet. Mutations in the FGFR2 gene cause Apert syndrome.

     

    Anderson et al report that in Apert's syndrome there is widespread anomalies of the feet, with defects including both predictable dysmorphic changes and progressive fusions of the skeletal components during skeletal maturity.

     

    Incorrect Answers:

    2: Achondroplasia is related to abnormalities in the FGFR3, not FGFR2.

    3: SED congenita is caused by mutations in COL2A1 (type II collagen alpha 1 chain) on chromosome 12. These result in abnormal type II collagen.

    4:The X-linked form of SED tarda is caused by mutation in SEDL (SED late)

    gene.

    5: Marfan syndrome is caused by defects in the fibrillin gene.

     

     

     

  78. What is the most important measure to take to reduce the risk of frostbite of the toes while hiking in extreme temperatures?

    1. Stop often for recovery breaks.

    2. Drink enough warm liquids.

    3. Reduce thermal heat loss from shoes.

    4. Use triple socks.

    5. Adequately "carbo load" before the start.

     

    CorreCt answer: 3

     

    Several studies showed the most reliable method to reduce the risk of cold exposure injury is to reduce thermal heat loss. This can be done with a combination of protective socks and shoes, and reducing moisture in the shoes.

     

     

     

  79. Figures 45a through 45e are the MRI scans, gross specimen, and histology of the specimen of a 19-year-old man who has an enlarging mass in the second interspace. He reports forefoot pain that is worse with athletic activity. Radiographs show erosive changes of the third metatarsal head. What is the most common complication associated with incomplete excision?

     

     

     

     

     

     

     

     

     

     

     

     

     

     

    1. Metastatic disease

    2. Malignant degeneration

    3. Recurrence

    4. Pathologic fracture

    5. Infection

    CorreCt answer: 3

     

    Giant cell tumor of the tendon sheath often arises from the synovial lining of tendon sheaths. This lesion is frequently found in the hand and foot. The lesion is slow growing and can invade adjacent structures. In the foot, wearing shoes or increased activity can cause pain. Incomplete or piecemeal excision can lead to recurrence.

     

     

     

  80. A 42-year-old construction worker presents with pain in his right wrist. A current radiograph of the wrist is shown in Figure A. He reports that rotating activities, such as turning a screw driver, are bothersome and the pain is preventing him from working. A current MRI reveals a TFCC tear, and nonsurgical treatment has failed to provide relief. Treatment should now consist of:

     

     

     

     

    1. Repair of the ulnar styloid nonunion

    2. Darrach resection of the distal ulna

    3. Complete ulnar head resection

    4. Ulnar hemiresection arthroplasty and TFCC reconstruction/repair

    5. Isolated arthroscopic TFCC reconstruction

      CORRECT ANSWER: 4

      The clinical presentation is consistent with DRUJ arthritis in a heavy laborer. Of the options listed, ulnar hemiresection arthroplasty with concurrent TFCC reconstruction or repair would be the most appropriate treatment.

       

      While there are multiple treatment options, the ulnar hemiresection arthroplasty with concurrent TFCC reconstruction or repair is considered most appropriate in heavy laborers, as it would likely resolve the pain and enable them to return to work sooner. The TFCC should be intact when performing an ulnar hemiresection arthroplasty to prevent distal ulna instability with forearm rotation. One could also consider performing a Suave-Kapandji procedure. This procedure creates a distal radioulnar fusion and an ulnar pseudarthrosis proximal to the fusion site through which rotation can occur. The advantage is that the ulnocarpal joint is not sacrificed, and a stable wrist is created.

       

      Scheker et al reported on the outcome of ulnar shortening performed on 32 wrists with early osteoarthritis of the DRUJ. The postoperative wrist ratings were 7/32 excellent, 11/32 good, 9/32 fair, 5/32 poor, with plate irritation being the most frequent postoperative complication.

       

      Figure A is a radiograph showing significant DRUJ arthritis. Illustration A shows ulnar hemiresection arthroplasty. Illustration B shows a Darrach procedure.

      Illustration C shows a Sauve-Kapandji procedure. Illustration D is a treatment schematic of TFCC reconstruction.

       

      Incorrect Answers:

      Answer 1: There is no obvious ulnar styloid non-union.

      Answer 2: As mentioned in Miller's review text, the Darrach procedure is typically reserved for low-demand, elderly patients and may lead to painful proximal ulna stump instability.

      Answer 3: Complete ulnar head resection is not indicated.

      Answer 5: TFCC reconstruction will not improve or treat the DRUJ arthritic changes.

       

       

       

       

       

       

       

       

       

       

       

       

  81. Figures 113a and 113b are the radiographs of a 7-year-old girl who was evaluated for a visible elbow deformity by a foster parent. She thought the child fell, but her history was vague. On physical examination, a large prominence was seen over the posterolateral elbow, and the girl lacks the terminal 20 degrees of elbow extension. She has 75 degrees of elbow pronation and supination. She was nontender on examination. What is the most appropriate next treatment step?

     

     

     

     

    1. Child abuse workup

    2. Closed reduction

    3. Open reduction with possible osteotomy

    4. Observation CORRECT ANSWER: 4

    The most appropriate management of this condition is observation. The patient most likely has a congenital dislocation of the radial head, although this may also represent a posttraumatic deformity. The absence of findings on physical examination speaks against an acute injury. The appearance of the radial head reveals the typical findings of a congenital dislocation, namely the convex appearance of the proximal radial articular surface. These children typically have very functional range of motion and do not require treatment unless they are symptomatic. There is nothing in this child's history to suggest abuse.

     

     

  82. The most common mechanism of injury to the triangular fibrocartilage complex (TFCC) involves

    1. wrist extension and forearm pronation.

    2. wrist extension and forearm supination.

    3. wrist flexion and forearm pronation.

    4. wrist flexion and forearm supination.

    5. axial load in ulnar deviation.

     

    CorreCt answer: 1

     

    TFCC tears are common in athletes. As the athlete braces for a fall, the wrist is most commonly in an extended position and the forearm is pronated.

     

     

     

  83. A 28-year-old woman fell on her right wrist while rollerblading 6 days ago. She was seen in the emergency department at the time of injury and was told she had a sprain. Examination now reveals dorsal tenderness in the proximal wrist but no snuffbox or ulnar tenderness. Standard wrist radiographs are normal. What is the next most appropriate step in management?

    1. Arthroscopy of the wrist

    2. CT of the wrist

    3. Bilateral PA clenched fist radiograph

    4. Electromyography and nerve conduction velocity studies

    5. AP and lateral radiographs of the forearm

    CORRECT ANSWER: 3

    When considering the diagnosis of scapholunate ligament injury, standard radiographic views of the hand will not always reveal widening of the scapholunate gap. Although MRI may reveal injury to the ligaments, the PA clenched fist view can be obtained in the office during the initial patient visit. Arthroscopy is not a first-line diagnostic tool.

     

     

     

  84. Figures 12a through 12c show the radiographs of a 28-year-old professional baseball player who has ulnar-sided wrist pain and

    numbness and tingling in the fourth and fifth digits for the past 6 weeks. Management should consist of

     

     

     

     

     

     

     

     

     

    1. cast immobilization.

    2. bone stimulation and splinting.

    3. ulnar nerve exploration.

    4. open reduction and internal fixation.

    5. excision of the fragment.

     

    CorreCt answer: 5

     

    Hook of the hamate fractures typically occur as a result of direct force from swinging a bat, golf club, or racket. Pain is localized to the hypothenar eminence. The injury is best seen on a carpal tunnel view. CT will confirm the diagnosis. Chronic cases can be associated with neuropathy of the ulnar nerve. Excision of the hook through the fracture site usually yields satisfactory results, allowing the athlete to return to competition.

     

     

     

  85. A 40-year-old right-handed professional football player reports persistent right wrist pain after falling during a game 5 days ago. A radiograph is shown in Figure 21. Management should consist of

     

     

     

    1. immobilization in a short arm thumb spica cast.

    2. immobilization in a long arm thumb spica cast.

    3. arthroscopic repair and percutaneous pinning.

    4. open repair and percutaneous pinning.

    5. dorsal capsulodesis.

     

    CorreCt answer: 4

     

    The radiograph reveals an increased distance between the scaphoid and the lunate, which is indicative of scapholunate disassociation. A ring sign is also present, which represents the distal pole of the scaphoid viewed end on in a palmarly flexed position. In the acute setting, the scapholunate can be repaired. Open repair and percutaneous pinning is the treatment of choice. Dorsal capsulodesis is performed in the chronic setting if such an injury is initially missed.

     

     

     

  86. An 18-year-old rugby player has had pain in his ring finger after missing a tackle 1 week ago. Examination reveals tenderness in the distal palm, and he is unable to actively flex the distal interphalangeal (DIP) joint. Radiographs are normal. What is the most appropriate management?

    1. Acute tendon repair

    2. DIP joint extension splinting for 6 weeks

    3. DIP and proximal interphalangeal joint extension splinting for 6 weeks

    4. Buddy taping to the middle finger for 2 weeks

    5. Early range-of-motion exercises and return to play as pain permits

    CORRECT ANSWER: 1

    Flexor digitorum profundus rupture or “rugger jersey finger” often occurs in the ring finger after the player misses a tackle and catches the digit on the shirt of the opposing player. Surgical repair is required for zone I-type injuries.

     

     

     

  87. A 65-year-old right-hand-dominant man has a 5 year history of progressive right wrist pain. He relates spraining his wrist playing football in college, but otherwise has had no prior traumatic injury. He is a pack per day smoker. An AP radiograph of the wrist is shown in Figure A. Wrist immobilization, anti-inflammatory medications, and injections have failed to provide relief. Which appropriate surgical treatment option offers the lowest risk of postoperative complications?

     

     

     

     

    1. Radial styloidectomy

    2. Total wrist arthroplasty

    3. Proximal row carpectomy

    4. Scaphoid excision with four-corner fusion

    5. Complete radiocarpal arthrodesis

    CORRECT ANSWER: 3

    Proximal row carpectomy (PRC) and scaphoid excision with four-corner fusion are both appropriate surgical treatment options for stage II scapholunate advanced collapse (SLAC) wrist; however PRC is associated with fewer postoperative complications, particularly in active smokers.

     

    Scapholunate interosseous ligament disruption leads to abnormal wrist biomechanics and degenerative arthritis. This progression follows a predictable pattern termed scapholunate advanced collapse. In stage II disease where the entire radioscaphoid articulation is affected but the capitolunate articulation is spared, both proximal row carpectomy (PRC) and scaphoid excision with four-corner fusion offer long-term pain relief while preserving wrist motion and grip strength. Scaphoid excision with four-corner fusion has a higher rate of complications owing to nonunion, hardware issues, and dorsal impingement from malunion. PRC is not recommended in the setting of capitolunate arthritis (stage III).

     

    Tomaino, et al. retrospectively compared PRC and limited intercarpal arthrodesis with scaphoid excision (LWF) at a mean of 5.5 years postoperatively in 24 symptomatic SLAC wrists. They noted good pain relief, grip strength, and function in all but 3 patients having undergone PRC - one of whom required revision to wrist arthrodesis (these patients had symptomatic capitate arthrosis). They concluded that in wrists without capitolunate arthritis, PRC had the benefit of being technically easier to perform, did not require prolonged postoperative immobilization, and avoided the risk of nonunion associated with LWF; however it was not an appropriate surgical option in stage III SLAC wrists with capitolunate involvement.

     

    Strauch reviewed the evaluation and treatment of SLAC and SNAC (scaphoid nonunion advanced collapse) wrists. Treatment options for SLAC wrist include four-corner fusion, capitolunate arthrodesis, PRC, radial styloidectomy, wrist denervation, and complete radiocarpal fusion. Excision of the distal ununited scaphoid fragment is an additional option in the setting of SNAC wrist. He additionally highlights current controversies between PRC vs. four-corner fusion.

     

    Figure A shows an AP radiograph with stage II SLAC wrist. The entire radioscaphoid articulation is arthritic with sparing of the capitolunate surface.

    Illustration A shows the modified Watson classification of scapholunate advanced collapse.

     

    Incorrect Answers:

    Answer 1: Radial styloidectomy may provide relief in stage I SLAC wrist; however, it does not adequately address the arthritis throughout the radioscaphoid articulation seen in more stage II.

    Answer 2: Total wrist arthroplasty is not indicated in stage II SLAC wrist. Answer 4: While scaphoid excision with four-corner fusion is an appropriate treatment for stage II SLAC wrist, it has a higher rate of complications compared to PRC due to nonunion, hardware complications, and dorsal impingement. This is particularly relevant in this patient, who is a pack per day smoker.

    Answer 5: Complete radiocarpal arthrodesis would be indicated in the setting of pancarpal arthritis or in failure after PRC. It is not the best treatment option for stage II SLAC wrist.

     

     

     

     

     

     

  88. A 25-year-old male presents to the clinic with a painful, enlarging mass at the volar radial wrist. He initially noticed the mass 6 months ago after he hurt his wrist golfing. Figure A shows a clinical photograph of the patient's wrist. Radiographs are unremarkable. An ultrasound of the mass is shown in Figure B. Surgical excision is planned. Which of the following is the most appropriate type of resection and histologic finding?

     

     

     

     

     

     

    1. Intralesional excision; synovial cells with mucin accumulation

    2. Incision & drainage; polymorphonuclear cells

    3. Wide excision; histiocytes with frequent giant cells

    4. Marginal excision; synovial cells with mucin accumulation

    5. Intralesional excision; histiocytes with frequent giant cells

    CORRECT ANSWER: 4

    The patient presents with a volar wrist ganglion cyst. Surgical treatment consists of marginal excision. Histologic analysis demonstrates synovial cells with mucin accumulation.

     

    Ganglion cysts are the most commonly presenting masses in the hand. These cysts consist of a synovial cell lining filled with mucin. Dorsal wrist ganglion cysts originate from the scapholunate interval and are more common than volar wrist ganglions, which typically originate from the scapho-trapezio-

    trapezoidal joint articulation. Ganglion cysts can cause pain related to mass effect. Ultrasound can help differentiate these masses from vascular malformations or other tumors; ganglion cysts present as homogenous anechoic masses with well-defined borders.

     

    Mayerson, et al. reviewed the diagnosis and management of soft-tissue masses. They highlight the typical presentation of ganglion cysts, which wax and wane in size and transilluminate with a pen light. The authors concluded that MRI is diagnostic if there remains any uncertainly after history and clinical exam.

     

    Head et al compared surgical excision versus needle aspiration of 2,239 adult wrist ganglions in a meta-analysis of 35 studies. Surgical excision resulted in a 76% reduction in recurrence compared to aspiration. Mean recurrence for arthroscopic excision (6%), open surgical excision (21%) and aspiration (59%) and mean complication rate for arthroscopic excision (4%) open surgical excision (14%) and aspiration (3%) were also determined. Data from arthroscopic excision was limited but is a promising technique. Open surgical excision has a significantly lower recurrence rate as compared to aspiration.

     

    Figure A shows a clinical photo of a volar wrist ganglion cyst. Figure B shows the ultrasound image of a volar wrist ganglion cyst.

     

    Incorrect Answers:

    Answer 1: Intralesional (incomplete) excision of a ganglion cyst has a high risk of recurrence.

    Answer 2: This describes an abscess.

    Answers 3 and 5: Histiocytes with frequent giant cells describes giant cell tumors of tendon sheath.

     

     

     

     

     

  89. A 27-year-old man falls on his hand at work. He notices an immediate deformity of his ring finger. Radiographs are provided in Figure A. Which of the following is the most appropriate initial treatment?

     

     

     

     

    1. Closed reduction, buddy taping, and early motion to prevent stiffness

    2. Closed reduction and full time extension splinting

    3. Open reduction and repair of the central slip of the extensor tendon

    4. Open reduction and repair of the volar plate

    5. Amputation and immediate return to work

    CORRECT ANSWER: 2

    The radiograph demonstrates a volar PIP dislocation. The central slip of the extensor tendon is frequently ruptured and will lead to a boutonneire deformity if left untreated. The PIP must be immobilized in extension to allow the extensor mechanism to heal. Immobilization in extension should be maintained for 6 weeks to allow soft tissue healing. Open reduction and repair of the central slip would be the appropriate treatment for a developing boutonneire deformity that presents in a subacute or chronic time basis.

     

    Illustrations A and B demonstrate a schematic and clinical photo of central slip disruption and secondary deformity with PIP flexion and DIP hyperextension (Boutonniere Deformity).

    Posner et al reviewed 7 patients with chronic palmar dislocations of the PIP joint who were treated with open reduction and reconstruction of the extensor mechanism. All patients acheived satisfactory range of motion and the authors concluded that this technique is preferable to arthrodesis.

     

    Peimer et al reviewed 15 patients with palmar dislocations of the PIP joint. Twelve of the fifteen were evaluated on a delayed basis (average 11 weeks following injury) and underwent open reduction and surgical repair of the extensor tendon. Three of the fifteen were seen earlier following injury and were treated with closed reduction and pinning. All fifteen patients acheived satisfactory clinical outcomes although finger range of motion was not fully recovered in any case.

     

     

     

     

     

     

     

     

  90. Figure A is of a 22-year-old male college basketball player presents for evaluation of a right index finger deformity. He reports a fall during a game 8 weeks ago, with resultant deformity to the index finger. He "popped it back in" and returned to play. Physical exam is most likely to demonstrate:

     

     

     

    1. Inability to passively extend the PIP joint to neutral, able to passively flex and extend the DIP joint

    2. With the PIPJ flexed, resistance to PIPJ extension causes the DIPJ to become supple

    3. Dorsal subluxation of the PIP joint, able to passively flex and extend DIP joint

    4. With the PIPJ flexed, resistance to PIPJ extension causes the DIPJ to become rigid

    5. Inability to actively flex the DIP joint, able to actively flex the PIP and the MCP joints

    CorreCt answer: 4

     

    The patient presents with a Boutonniere deformity secondary to a traumatic central slip disruption in the setting of volar PIP joint dislocation. Physical exam will demonstrate a positive Elson's test, which is described in answer 4.

     

    The digital extensor mechanism consists of the central slip and two lateral bands, all of which arise from the extensor digitorum communis (EDC) tendon. Flexion of the PIP joint puts the central slip on tension, and volarly subluxes the lateral bands causing them to become slack. Tension on the central slip causes extension of the PIP joint, with concomitant dorsal shift of the lateral bands which help to bring the DIP joint into extension.

     

    In 1986, Elson described his physical examination maneuver for diagnosis closed rupture of the central slip. With the hand resting on the edge of a table, the PIP joint is flexed to 90 degrees over the table edge, and the patient is asked to extend the digit against resistance. Active extension of the middle phalanx can only be observed with an intact central slip, and the adjacent lateral bands will remain slack which allows the DIP joint to remain flail. In central slip ruptures, effort to extend the middle phalanx will be accompanied

    by DIP rigidity/extension as the lateral bands are forced to contribute to extension.

     

    Rubin et. al. performed a cadaveric study evaluating the efficacy of physical examination maneuvers to identify acute ruptures of the central slip. They

    found that Elson’s test was the only maneuver that could discern central slip integrity in both tested scenarios: 1) pre-boutonniere deformity with division of the central slip and 2) passively correctible boutonniere deformity caused by division of the central slip, the triangular ligament, and the oblique fibers of the extensor expansion.

     

    Figure A is a clinical image of an index finger with boutonniere deformity. Video A is a short demonstration of how to perform the Elson test.

    Incorrect answers:

    Answer 1: Inability to passively extend the PIP joint is also known as a pseudo-boutonniere deformity and is common after dorsal PIP dislocations which involve injury to the volar plate

    Answer 2: With resistance to PIPJ extension, the DIPJ is supple - describes a normal finger during an Elson's test.

    Answer 3: Dorsal subluxation of the PIP joint would be seen following dorsal PIP joint dislocations

    Answer 5: Inability to actively flex the DIP joint would occur with rupture of the flexor digitorum profundis

     

     

     

  91. A 25-year-old woman presents to the clinic after knife injury to the volar aspect of her long finger 2 weeks ago. She is evaluated and diagnosed with tendon rupture of the flexor digitorum profundus (FDP). What finding on examination can be expected in this patient?

    1. With passive wrist extension, extension remains at the distal interphalangeal joint

    2. With passive wrist extension, extension remains at the proximal interphalangeal joint

    3. With passive wrist flexion, extension is limited at the distal interphalangeal joint

    4. With passive wrist flexion, flexion remains at the distal interphalangeal joint

    5. With passive wrist flexion, flexion remains at the proximal interphalangeal joint

    CorreCt answer: 1

     

    With an FDP rupture, physical exam would likely reveal loss of flexion at the DIP joint both actively and passively with wrist extension.

     

    When the wrist is in extension, flexor tendons are stretched and should result in flexion at the DIP (FDP) and PIP (FDS) joints. The FDP tendon is responsible for flexion of the DIP joint, and this joint would remain extended during normal tenodesis on passive wrist exam. Inversely, with extensor tendon injuries, there may be a loss of digit extension with passive wrist flexion.

     

    Strickland presents a review article (Part 1) on flexor tendon injuries discussing clinical presentation and repair techniques. A commonly tested concept is that tendon repair is proportional to the number of core sutures, and currently recommended repair includes at least 4 core sutures for strength with epitendinous suture to aid in gliding and provide some strength.

     

    Kamal et al. present current evidence regarding flexor tendon injuries, reviewing examination, repair, and rehab. They note that to date there still remains heterogeneity in treatment patterns and no clear standard of care. Rehab options include no motion, early active range of motion, and controlled passive range of motion. The authors note that early loading may lead to improved strength.

     

    Illustration A depicts the usual tenodesis effect of the digits where passive extension of the wrist produces flexion of the fingers.

     

    Incorrect Answers:

    Answer 2: The FDP tendon acts at the DIP, not PIP joint.

    Answers 3, 4, and 5: With wrist flexion, there is no loading or stretch of the flexor tendons.

     

     

     

     

     

  92. A 20-year-old college football lineman sustains an injury to his index finger during a game. A radiograph of the hand is demonstrated in Figure A. What is the mechanism of injury and most common reason for unsuccessful closed reduction?

     

     

     

     

    1. Hyperextension mechanism causes the metacarpal head to button hole between the flexor tendon and the lumbrical

    2. Hyperextension mechanism causes volar plate avulsion and entrapment dorsal to the metacarpal head

    3. Rotational mechanism causes the metacarpal head to button hole between the flexor tendon and the lumbrical

    4. Hyperflexion mechanism causes volar plate avulsion and entrapment dorsal to the metacarpal head

    5. Hyperflexion mechanism causes the metacarpal head to button hole between the flexor tendon and the lumbrical

    CorreCt answer: 2

     

    Irreducible dorsal metacarpophalangeal (MP) joint dislocations occur from a hyperextension moment, which causes volar plate displacement and incarceration dorsal to the metacarpal head.

    MP joint dislocations are most commonly dorsal and occur with hyperextension injuries. Simple dislocations are reducible with wrist flexion (to relax the intrinsic muscles) and direct palpation over the proximal phalanx base.

    Complex dislocations occur with interposition of the volar plate. When irreducible, open reduction is required.

     

    Afifi et al. performed a cadaver study defining the anatomy surrounding irreducible dorsal index MP joint dislocations. They found that of all local structures, only release of the volar plate allowed for reduction of the MP joint. They concluded that volar plate interposition dorsal to the metacarpal head was responsible for irreducible MP joint dislocations.

     

    Bohart et al. describe 9 patients with irreducible dorsal MP joint dislocations (5 thumbs and 4 index fingers). A dorsal approach was performed in each case to allow for reduction of the volar plate. A stable MP joint was achieved in each case. They advocate for a dorsal approach, which minimizes the risk of iatrogenic injury to the neurovascular bundles, which are displaced volarly by the metacarpal head.

     

    Figure A shows an oblique radiograph of the hand demonstrating a dorsal dislocation of the index MP joint. Illustration A provides a schematic of both a simple and a complex dorsal MP joint dislocation. In the case of a complex dislocation, the volar plate avulses from its origin and becomes entrapped dorsal to the metacarpal head.

     

    Incorrect Responses:

    Answer 1, 3, 4, 5: Dorsal MP joint dislocations typically occur with hyperextension injuries and can become irreducible with volar plate entrapment dorsal to the metacarpal head. The mechanism is not typically hyperflexion or rotation. The flexor tendon and the lumbrical do not contribute to irreducible dorsal MP joint dislocations.

     

     

     

     

     

  93. A 3-year-old patient presents to clinic with her parents for the chest wall anomaly seen in Figure A. What other congenital disorder is associated with this syndrome?

     

     

     

     

    1. Flexible pes planovalgus

    2. Syndactyly

    3. Polydactyly

    4. Macrodactyly

    5. Accessory navicular CORRECT ANSWER: 2

    The figure shows an individual with Poland's Syndrome, as demonstrated by the absent sternoclavicular head of the pectorals major. Syndactyly and symbrachydactyly is often seen, in addition to hypoplasia and shortening of the fingers.

     

    Poland's Syndrome, or Poland anomaly/sequence, is thought to be caused by disruption of the subclavian artery in utero, causing various hypoplastic anomalies of the upper extremity. These are typically ipsilateral ranging from aplasia of the sternocostal head of the pectorals major, radio-ulnar synostosis, symbrachydactyly and other limb hypoplasias, or syndactyly of the central digits. Syndactyly is often simple and either complete or incomplete. It is addressed surgically early on, with the chest wall deformities needing reconstruction and muscle transfers closer to sexual maturity. Thoracic, cardiovascular, and genitourinary anomalies may also be present.

    Catena et al. proposed a new classification system for Poland Syndrome based on the degree of clinical severity of the entire upper extremity. The classification type increased with more proximal involvement up the upper extremity. This new system may help guide treatment as is takes into account the functional state of the rest of the upper extremity and not just the hand, as previous systems have.

     

    Ireland et al. analyzed 43 consecutive cases of Poland's Syndrome. All cases involved congenital aplasia and syndactyly which was typically simple and incomplete. The thumb can be involved putting it the same plane as the fingers. Anomalies were more frequently seen on the right side. They noted favorable outcomes with surgical correction by syndactyly release initiated by 1 year, with some requiring periodic revision releases, while others required an amputation producing a three-fingered hand.

     

    Figure A shows an absent stenocostal head of the pec major. Only the right side is involved. Illustrations A-C show pre-op and post-op digital release of an individual with syndactyly

     

    Incorrect Answers:

    Answers 1 and 5: Poland syndrome has no effect on the lower extremity. Answers 3 and 4: Polydactyly and macrodactyly have not been seen in Poland Syndrome.

     

     

     

     

     

     

     

     

     

     

  94. A 32-year-old man sustains an injury to his left thumb. Examination in the ER demonstrates a 2x4 cm wound on the dorsal thumb overlying the proximal phalanx with exposed tendon and bone. What is the most appropriate option for soft tissue coverage?

    1. Cross-finger flap

    2. Moberg advancement flap

    3. Full-thickness skin graft

    4. First dorsal metacarpal artery flap

    5. V-Y advancement CORRECT ANSWER: 4

    The first dorsal metacarpal artery flap (Kite flap) is the most appropriate soft tissue coverage option for dorsal thumb wounds that disrupt vascularized tissue overlying the extensor tendon and bone (including the epidermis, dermis, subcutaneous tissue, and tenosynovium) when primary closure is not possible.

     

    Kite flaps are based off of the first dorsal metacarpal artery, which overlies the index finger metacarpal. It offers a pedicle length up to 7 cm and can reliably cover soft tissue defects up to 3x5 cm in area. Given its location, it is appropriate for the treatment of thumb wounds including those to the web space, dorsum, and volar pulp, particularly when injury compromised the vascularity of the wound bed. It can be modified to include both dorsal branches of the proper digital nerve, thereby conferring sensibility to the covered wound. The donor site can subsequently be covered with a full-thickness skin graft.

    Rehim et al. reviewed local flaps of the hand. They offer treatment options and appropriate indications based upon the anatomic location and size of the wound within the hand. They conclude that when there are no clinical limitations, local flaps provide ideal soft tissue coverage and function for hand wounds based upon the local anatomy without the need for more complex free tissue transfers.

     

    Eberlin et al. review soft tissue coverage options in the hand. They present four clinical cases and offer one established and one non-traditional surgical treatment option for each. They recommend the first dorsal metacarpal artery flap as an established treatment option in a case of thumb volar pulp injury as it offers contour restoration as well as sensibility when the digital nerves are included with the vascular pedicle.

     

    Illustration A demonstrates a large dorsal thumb soft-tissue injury that is treated with first dorsal metacarpal artery flap coverage and full-thickness skin grafting to cover the donor site.

     

    Incorrect Answers:

    Answer 1: Cross-finger flaps are indicated to treat volar soft tissue defects of the finger with intact overlying tenosynovium.

    Answer 2: Moberg advancement flaps are indicated for the treatment of volar thumb pulp injuries < 2 cm.

    Answer 3: Full-thickness skin grafts are not indicated in the presence of exposed bone or tendon where the overlying vascularized tissue (tenosynovium) has been disrupted.

    Answer 4: V-Y advancement is a treatment option for closure of finger tip amputations.

     

     

     

     

     

     

  95. A 65-year-old man complains of numbness and tingling in the thumb, index, and long fingers of his dominant right hand for 3 months. An EMG demonstrates prolonged median sensory latency and low amplitude compound muscle action potentials with fibrillations in the abductor pollicis brevis. What is the most appropriate treatment option and the rate of continued symptoms at 1 year after treatment?

    1. Splinting and corticosteroids; 5%

    2. Open carpal tunnel release; 20%

    3. Splinting and corticosteroids; 30%

    4. Endoscopic carpal tunnel release; 2%

    5. Open carpal tunnel release; 5%

    CORRECT ANSWER: 2

    The most appropriate treatment of carpal tunnel syndrome (CTS) with EMG evidence of denervation is surgical release. The rate of residual symptoms at 1 year is approximately 20%.

     

    The American Association of Electrodiagnostic Medicine (AAEM) criteria delineates CTS severity by EMG. Mild CTS is purely sensory. Moderate disease demonstrates prolonged sensory and motor latencies. Severe disease progresses to involve muscle denervation. Mild and moderate CTS may be treated with carpal tunnel release following failure of nonoperative treatment; however, early operative treatment is supported for severe disease to limit further denervation. Patients experience significant improvement in

    symptoms; however, recovery is prolonged and persistent symptoms may be present in ~20% at 1 year.

     

    Kronlage et al. compared changes in numbness and pain following carpal tunnel release in 47 patients with moderate and 48 patients with severe CTS diagnosed on EMG. At 1 year or longer, 1 (2%) patient with moderate disease had continued symptoms compared to 9 (19%) of patients with severe CTS. They concluded that patients with severe CTS experience significant reductions in symptoms following carpal tunnel release; however, recovery may be prolonged or incomplete at 1 year postop.

     

    Ono et al. performed a systematic review of 25 studies reporting outcomes for the treatment of carpal tunnel syndrome. They noted an increasing trend towards recommending earlier surgery for CTS with or without median nerve denervation. They conclude that this differed from the 2007 AAOS guidelines, which recommended early surgery only in the setting of muscle denervation.

     

    Incorrect Answers:

    Answer 1, 3: The patient has muscle denervation on EMG. This is severe CTS and surgical intervention is indicated. Persistent symptoms occur in ~20% of patients despite surgical intervention at 1 year postop.

    Answer 4, 5: Surgical intervention is warranted; however, persistent symptoms occur in ~20% of patients at 1 year.

     

     

     

  96. A 23-year-old man presents with chronic, progressive right wrist pain. He remembers falling onto an outstretched hand 2 years ago. Radiographs, CT scans and a T1-weighted coronal MRI are shown in Figures A through E. No bleeding was identified at surgery. In addition to surgical stabilization, what is the next best step?

     

     

     

     

     

     

     

     

     

     

     

     

     

    1. Corticocancellous autograft inserted through a dorsal approach

    2. Pedicled distal radius graft inserted through a dorsal approach

    3. Pedicled distal radius graft inserted through a volar approach

    4. Free vascularized femoral bone graft inserted through a dorsal approach

    5. Free vascularized femoral bone graft inserted through a volar approach

    CORRECT ANSWER: 5

    This patient has an old scaphoid waist fracture with nonunion, proximal pole avascular necrosis (AVN), and carpal collapse. Optimal treatment is with a free vascularized medial femoral condyle (MFC) graft through a volar approach.

     

    Where there is proximal pole AVN, union was achieved in 88% of patients with a vascularized graft versus 47% with screw and nonvascularized wedge bone graft fixation. The 1,2 intercompartmental supraretinacular artery (1,2 ICSRA) pedicle graft leads to union rates of 71% for scaphoid nonunions and 50% for AVN. The risk for failure is higher when there is DISI or humpback deformity (underscoring the need to restore scaphoid geometry). The MFC graft uses a pedicle from the descending genicular artery or the superomedial genicular artery when the descending genicular artery is not present. The volar approach is preferred as it allows correction of the humpback deformity and anastomosis of the MFC pedicle to the radial artery.

     

    Jones et al. retrospectively compared 2 vascularized bone grafts for treatment of scaphoid waist nonunions with proximal pole AVN and carpal collapse. 4 of

    10 nonunions treated with distal radial pedicle graft healed at 19 weeks. 12 of 12 nonunions treated with free vascularized medial femoral condyle (MFC) graft healed at median of 13 weeks. Rate of union was higher, and time to healing was shorter for the MFC graft. They recommend the MFC vascularized bone graft for treatment of scaphoid waist nonunion with proximal pole AVN and carpal collapse.

     

    Figures A, B, C and D are PA and lateral radiographs and coronal and sagittal CT images showing scaphoid waist nonunion with carpal collapse and osteonecrosis of the proximal pole, respectively. Figure E is a T1-weighted coronal image shows diffusely decreased signal within the proximal pole.

    Illustrations A and B show harvest and inlay of the 1,2 ICSRA graft. Illustration C shows the MFC graft.

     

    Incorrect Answers:

    Answers 1, 2: Nonvascularized grafts are not recommended. A volar approach is used for nonunions with humpback deformity.

    Answers 3, 4: The 1,2 ICSRA graft is taken from the dorsal aspect of the distal radius (dorsal approach). This graft has inferior union rates compared with the MFC graft. The dorsal approach is necessary because the 1,2 ICSRA pedicle is located dorsally.

     

     

     

     

     

     

     

     

     

     

  97. A 38-year-old female develops pain and pallor in all the digits of the right hand daily. Her symptoms have progressed over 2 years despite avoiding direct cold exposure and multiple medications including nifedipine. Recently she has developed the lesions seen Figure A. Workup for underlying disease by her rheumatologist was negative. She is a candidate for Botuninum toxin A injections. What is the physiologic effect of botulinum toxin in the hand for her condition?

     

     

     

     

    1. Improving proprioception in the fingers and hand by binding to postsynaptic acetylcholine receptors

    2. Improving digital perfusion by cleaving pre-synaptic SNAREs and preventing the release of acetylcholine

    3. Decreasing glabrous skin sensation by reducing hyperexcitability of voltage dependent calcium channels

    4. Strengthening the intrinsic muscles by increasing hyperexcitability of voltage dependent calcium channels

    5. Increasing sympathetic innervation by cleaving pre-synaptic SNAREs and preventing the release of acetylcholine

    CorreCt answer: 2

     

    The patient is displaying Raynaud's Disease with the development of ulcerations from chronic vasoconstriction. Botulinum toxin has been shown to increase the blood supply throughout the hand through the its well-known mechanism of pre-synaptic SNARE cleavage.

    Botulinum toxin cleaves the pre-synaptic SNAREs (soluble NSF attachment potion receptor) and prevents the release of acetylcholine from the intracellular vesicles. This has been used for multiple medical purposes, including vasospastic disorders. Raynaud's Disease is characterized by idiopathic vasospasm of the digital arteries without known underlying cause. Usually afflicting pre-menopausal women, it begins with pain and pallor in the digits, typically affecting the bilateral hands. Avoiding cold environments and tobacco are the mainstays of treatment, with calcium-channel blockers being the most common medication used. When these and other medications fail, botulinum toxin injections have been shown to be of benefit by relieving vasoconstriction and decreasing ischemia and pain.

     

    Neumeister et al. reviewed the application of botulinum toxin A and individuals with Raynaud's Disease and Syndrome. They showed marked increases (up to 300%) in digital perfusion in patients receiving these injections into the common digital vessel at the level of the palm. They concluded the mechanisms for this response are likely multifactorial, involving central and systemic effects on neurotransmitters involved in chronic pain pathways, local digital vessel tone, and sympathetic innervation.

     

    Iodio et al. reviewed all clinical studies regarding the use of bootulinum toxin A in raynaud's. There was high variability among the studies in terms of dosage and application method, but all studies reported favorable patient outcomes and some showed improved healing of ulcerations. These studies are promising but are limited due to study design and lack of standardization of botulinum toxin application.

     

    Figure A shows non-infected ulcerations in the digits, common in progressive raynaud's disease.

    Illustration A shows ischemic digits due to Raynaud's Syndrome. Illustration B is the same hand after botulinum toxin A injection. Illustration C is a laser doppler of a hand both pre- and post-injection perfusion of botulinum toxin A in an individual with Raynaud's Disease. Illustration D depicts the recommended method of injection, placing 10 units of botulinum around the common digital vessel at the level of the palm.

     

    Incorrect Answers: There is no shown effect on digital proprioception, skin sensation, muscle strengthening.

    Answer 1: This is the mechanism of non-depolarizing neuromuscular blocking agents used in anesthesia

    Answers 3 and 4: Reduing excitability of calcium channels is the mechanism through which of gabapentin and pregabalin function

    Answer 5: Though not confirmed, botulinum toxin may directly or indirectly decrease sympathetic innervation to the digits

     

     

     

     

     

     

     

     

     

     

     

     

     

     

  98. A 27-year-old male injures his thumb during a fall onto an outstretched hand. He has pain at the MCP joint and difficulty grasping objects between the thumb and index finger. He undergoes surgery with the planned incision shown in Figure A. What muscle and corresponding nerve innervates the structure that blocks reduction of the ligament shown in Figure B?

     

     

     

     

     

     

     

    1. Opponens pollicis, median nerve

    2. Flexor pollicis brevis, ulnar nerve

    3. Adductor pollicis, ulnar nerve

    4. Abductor pollicis brevis, median nerve

    5. Adductor pollicis, median nerve

    CORRECT ANSWER: 3

    The patient has an ulnar collateral ligament injury. The structure that blocks reduction of the ligament is the adductor pollicis aponeurosis, which is innervated by the ulnar nerve.

     

    Thumb ulnar collateral ligament injuries occur after a radially directed force on an extended thumb, stressing the ulnar collateral ligament, dorsal capsule and volar plate. The thumb should be radiographed before stress exam if the history warrants so as not to displace a possible bony avulsion. Exam includes valgus stress on the thumb at 0 and 30 degrees of flexion to test the accessory and proper collateral ligaments respectively. With complete rupture of both ligaments, a bump over the ulnar thumb MCP joint may be palpated, signifying a Stener lesion. The ligament usually tears at the distal insertion and displaces proximal and superficial to the adductor aponeurosis. The dorsal capsule and volar plate may also be injured.

     

    Bean et al. evaluated the biomechanics of non-anatomic reconstruction of the ulnar collateral ligaments in cadaveric specimens. They showed that 2mm of volar displacement of the ligament origin will allow for 10 degrees more radial deviation than anatomic placement will. This highlights the need for anatomic reconstruction and that deviation from this will alter joint kinematics.

     

    Figure A shows a planned incision over the ulnar aspect of the thumb MCP joint

    Fibure B shows a Stener lesion that is migrated proximally compared to the aponeurosis which is marked by the forceps

    Illustration A depicts the retraction of the collateral ligament proximal to the aponeurosis

    Illustration B shows a T1 MR coronal image showing a distal avulsion of the UCL and the Stener lesion, denoted by the asterisk, and the arrow pointing to the aponeurosis

     

    Incorrect Answers:

    Answer 1, 2 and 4 are all thenar muscles which are innervated by the median nerve, however they do not play a role in Stener lesions

    Answer 5. The adductor pollicis is innervated by the ulnar nerve

     

     

     

     

     

     

     

     

  99. An otherwise healthy 5-year-old female is brought to your office for the deformity shown in Figures A and B. Only the small digit of the left hand is involved and it may be fully flexed, but there is limited passive extension. What is the next best step in treatment?

     

     

     

     

     

     

    1. Observation and reassurance

    2. Nighttime extension splinting and stretching regimen

    3. Nighttime extension splinting and stretching regimen with full genetic workup

    4. FDS transfer to radial lateral band

    5. FDS split with transfer of limbs to A2 pulley and central tendon hood

    CORRECT ANSWER: 2

    The patient described has isolated camptodactyly with a mild flexion contracture. The best next step in treatment is to begin a stretching and splinting regimen.

     

    Camptodactyly is a nontraumatic flexion deformity isolated to the proximal interphalangeal joint, typically involving the small finger. This is often seen

    bilaterally and sporadically, although many congenital disorders are associated. Many underlying anatomical structures have been implicated in the pathogenesis of this condition, with various surgical techniques having been described to address these. If this condition remains untreated, adjacent joint involvement can develop, with MCP hyperextension seen most commonly.

    Intrinsic-plus splinting of the hand with passive stretching exercises should be initiated first. Surgery is usually reserved in cases of failed splinting or significant contractures approaching 60 degrees.

     

    Comer et al. reviewed the complications of campylodactly. Most common complications were progression or failed improvement of both PIP contracture and MP hyperextension, isolated PIP postoperative residual stiffness, and bony remodeling of proximal phalanx head preventing full extension. They note inconsistent results after surgical correction which supports early detection and conservative modalities as the mainstay of treatment, focusing heavily on a stretching program and night splinting.

     

    Rhee et al. reviewed outcomes of passive stretching for isolated camptodactyly flexion contractures in a series of children under the age of three years. They showed marked improvement of contracture deformity in all children across all levels of severity, though to less extent with more severe deformities.

     

    Figures A and B demonstrate early contracture of the left small finger. Illustration A is a radiograph showing maintenance of articular congruity.

     

    Incorrect Answers:

    Answer 1- Splinting and stretching have shown better results with shorter duration of deformity so beginning treatment would be optimal

    Answer 3- Though many congenital disorders, such as arthrogryposis, have been implicated in cases of camptodactyly, there is no evidence of this as this is isolated and unilateral.

    Answers 4 and 5- These are described methods for correcting camptodactyly, though conservative treatment should be sought first.

     

     

     

     

     

  100. A collegiate rower complains of dorsal wrist pain for 6 weeks refractory to NSAIDs and bracing. Maximal tenderness is palpated on the dorsoradial forearm approximately 5 cm proximal to the wrist. Pain is exacerbated with resisted wrist extension. Radiographs are unremarkable. A steroid injection should be directed into the compartment containing which of the following structures?

    1. APL and EPB tendons

    2. ECRL and ECRB tendons

    3. EPL tendon

    4. APL and ECRB tendons

    5. Brachoradialis tendon CORRECT ANSWER: 2

    The clinical scenario is consistent with intersection syndrome, a inflammatory response to overuse at the site of the second dorsal compartment crossing under the first dorsal compartment approximately 5 cm proximal to the wrist. An anatomical depiction is provided in illustration A. Injections of the second dorsal compartment, which includes ECRL and ECRB, may relieve symptoms

    and quell inflammation. Intersection must be differentiated from DeQuervain's syndrome, which is tenosynovitis of the first dorsal compartment. Injections of the first dorsal compartment, which includes APL and EPB, are part of the treatment algorithm for Dequervain's. Wood et al summarizes the evaluation and treatment of sports-related wrist injuries. Grundberg et al demonstrates the pathologic abnormality of intersection syndrome is stenosing tenosynovitis of the second compartment explaining the rationale behind steroid injections into the sheath.

     

     

     

     

     

     

  101. A 42-year-old chef has finally been transferred to the hand specialist 15 hours after injuring his non-dominant hand index finger with a butcher's knife as seen in figure A. He has kept the finger with him, which has been wrapped in saline-soaked gauze and placed on ice. What is the best reason the finger tip should not be replanted?

     

     

     

    1. The replanted digit will likely have poor function due to the delay in care

    2. Possible malingering

    3. The replanted digit will likely have poor function due to the local anatomy

    4. Patient age

    5. Workers compensation patients will have worse outcomes

    CORRECT ANSWER: 3

    Single digit amputations proximal to the insertion of the flexor digitorum superficialis (FDS), in generally have poor function and severe stiffness following replantation.

     

    Replantation between the FDS insertion and the distal palmar crease (zone 2 flexor tendon injuries) has historically led to poor results due to stiffness at the proximal interphalangeal joint, decreased sensation in the finger, and tendon adhesions between the FDP and slips of the FDS. Furthermore, outcome studies have demonstrated patients with index finger amputations through this region are more likely to bypass their stiff index finger and utilize their long finger for most tasks. However, amputation of multiple digits through zone 2 would be considered for replantation.

     

    Urbaniak et al performed a retrospective case series of 59 patients who

    underwent finger (thumb excluded) replantation for traumatic amputation. They found the functional results were most dependent on level of amputation and patients with amputation proximal to the insertion of the FDS had significantly decreased PIP motion. They concluded that replantation through zone 2 is seldom indicated due to severe stiffness.

     

    Boulas et al reviewed digital replantation and recommend initial treatment should consist of wrapping amputated parts in moistened gauze and placing on ice. Sharp and clean amputations are considered more viable candidates for replantation due to limited damage to the replantation junction compared to crush injuries. Additionally, they state the patients with major psychiatric disorders or those that are unable to comply with postoperative protocols should also be considered poor candidates for replantation.

     

    Figure A demonstrates an amputation through the left index finger proximal phalanx with no evidence of comminution or crush injury. Illustration A demonstrates the flexor tendon zones.

     

    Incorrect Answers:

    Answer 1: The general thresholds for cold ischemia time are 24 hours distal to the carpus and 12 hours proximal to the carpus.

    Answer 2: This amputation might be intentional and self-inflicted, however, the best reason not to replant is due to the poor function following replantation.

    Answer 4: Age is not a contraindication to replantation

    Answer 5: While workers compensation patients might report worse outcomes, the best reason not to replant is because of the historically poor outcomes of zone 2 replantations.

     

     

     

     

     

  102. A patient sustains an acute, closed injury to his index finger. The clinical appearance of the finger is shown in Figure A. The patient is asked to extend the finger against resistance, with the PIP joint in 90 degrees of flexion. You note that PIP joint extension was weak, with hyperextension and restricted passive flexion of the DIP joint. When planning to treat this injury non-operatively which active joint motion is encouraged?

     

     

     

     

     

     

    1. DIP flexion

    2. MCP flexion

    3. MCP extension

    4. PIP extension

    5. PIP flexion CORRECT ANSWER: 1

    This patient has sustained a central slip injury. Treatment consists of full time extension splinting of the PIP joint for 5 weeks with active DIP motion (flexion) encouraged.

     

    A central slip injury, or a zone 3 extensor tendon injury, is characterized by PIP flexion and DIP extension (boutonniere deformity). This is most often caused by a rupture of the central slip over the PIP joint caused by a laceration, a traumatic avulsion, or capsular distension in rheumatoid arthritis. A rupture of the central slip causes the extrinsic extension mechanism from the EDC to be lost and prevents extension at the PIP joint. This allows the lumbricals' pull to become unopposed, causing PIP flexion and DIP extension. The examination maneuver described in the question stem is the Elson Test. It is the most reliable way to diagnose a central slip injury before the deformity is present. Non-operative treatment may be undertaken if the injury is closed and presents acutely. The PIP is splinted in full extension for 5 weeks. Active DIP extension and flexion in the splint is encouraged to avoid contraction of the oblique retinacular ligament.

    Posner et al. describe the diagnosis and treatment of finger deformities following injuries to the extensor tendon mechanism. They suggest that treatment of a boutonniere deformity depends on its stage. For the acute injury (within the first 2 weeks), immobilization of the proximal interphalangeal joint in full extension for 5 weeks using a static splint that permits active and passive flexion of the DIP joint is usually effective.

     

    Figure A is a clinical photograph demonstrating an index finger with the classic boutonniere deformity of flexion at the PIP joint and hyperextension of the DIP joint. Figure B is a diagram showing the Elson test. When the central slip is intact, there is no hyperextension of the distal phalanx. When the central slip is disrupted, the distal phalanx can hyperextend due to the function of the tight lateral bands.

     

    Incorrect Answers:

    Answers 2, 3, 4, 5: These do not describe the active joint motion needed in the treatment of a closed central slip injury.

     

     

     

  103. A 25-year-old male is stabbed in the proximal volar forearm while fighting in a bar. He presents to the ED with a 1 cm wound and moderate oozing of blood. On exam, he has normal sensation throughout all distributions in his hand, normal radial and ulnar pulses, and a normal tenodesis effect. He is unable to actively flex his index finger DIP joint. Which muscle will also likely not function as a result of his injury?

    1. Flexor digitorum brevis

    2. Flexor carpi radialis

    3. Flexor carpi ulnaris

    4. Flexor pollicis longus

    5. Pronator teres CORRECT ANSWER: 4

    The patient has sustained a laceration of the anterior interosseous nerve (AIN), which is a branch of the median nerve and innervates the flexor pollicis longus, pronator quadratus, and the flexor digitorum profundus to the index and long fingers. An intact tenodesis effect signifies that all of his tendons are structurally intact.

     

    The AIN can be injured by a penetrating injury or chronic compression. It

    arises from the dorsoradial aspect of the median nerve distal to the elbow. It then passes between the FPL and FDP to lie on the anterior interosseous membrane en route to the pronator quadratus and wrist capsule (Illustration A). Compression sites of the AIN include the deep head of the pronator teres, FDS arcade, edge of the lacertus fibrosus, an accessory head of the FPL, or other accessory muscles of the forearm (FDS, FDP, FCR). In this particular scenario of an acute, penetrating AIN injury, exploration and primary end-to-end suture repair is appropriate.

     

    Rodner et al. review AIN syndrome and stress the importance of ruling out a tendon rupture, which can present similarly and can be differentiated by testing the patient's tenodesis effect. Non-traumatic AIN syndrome is usually the result of a neuritis, similar to Parsonage-Turner Syndrome (brachial plexus neuritis), and may have similar triggers such as viral infection or autoimmune disease. They recommend a prolonged period of observation (~12 months; in the absence of an obvious compressive or space-occupying lesion) due to high rates of spontaneous recovery at about one year.

     

    Park et al. report on 11 patients that underwent surgical exploration for spontaneous AIN syndrome at an average of 7.8 months. The most common compressive structure was a fibrous band of the FDS, however, four patients had no obvious compressive structure, emphasizing the importance of at least six months of conservative treatment.

     

    Incorrect answers:

    Answer 1: The FDB is a muscle on the plantar foot innervated by the medial plantar nerve.

    Answer 2: The FCR is innervated by the median nerve Answer 3: The FCU is innervated by the ulnar nerve

    Answer 5: The pronator teres is innervated by the median nerve prior to the departure of the AIN

     

     

     

     

     

  104. A 38-year-old female presents with 8 months of gradual weakness of her right hand. She denies paresthesias, numbness, and pain in the right upper extremity. She has compensatory thumb interphalangeal flexion during key pinch and intact two point discrimination. She has a negative Tinel's sign at the wrist and elbow. Electromyography (EMG) shows normal sensory conduction velocities but delayed motor conduction to the first dorsal interosseous muscle. Figure A and B show MRI images of pre and post contrast, respectively. Ultrasound is shown in Figure C. What is the next best step?

     

     

     

     

     

     

    1. Biopsy of the mass

    2. Cyst excision

    3. MRI of cervical spine

    4. Excision of the hook of hamate

    5. Cubital tunnel release CORRECT ANSWER: 2

    The patient has pure motor symptoms from ulnar nerve compression by a ganglion cyst at Guyon's canal. The next best treatment is excision of the ganglion cyst.

     

    Atraumatic compression of the ulnar nerve at Guyon's canal is caused by a ganglion cyst 80% of the time. Compression may present with mixed motor and sensory or pure motor symptoms. With purely motor compression the deep branch of the ulnar nerve is affected resulting in weakness of adductor pollicis. Subsequent loss of metacarpophalangeal flexion and adduction leads to a positive Froment's sign with compensatory thumb IP flexion. Pure motor compression will result in normal sensory examination and intact two point discrimination as sensory branches are unaffected. EMG will localize decreased velocities at the wrist. When neurologic symptoms are present, cyst excision is recommended. Ganglion cysts in this location often arise from the pisohamate joint and excision of the stalk is important to prevent recurrence.

     

    Wang et al. retrospectively investigated the outcomes of 9 patients with

    ganglion cysts with symptomatic compression of the deep branch of the ulnar nerve. At a mean follow-up of 23 months they found all patients had improved grip and tip pinch strength. They conclude that surgical intervention can lead to satisfactory outcomes.

     

    Shen et al review the imaging findings possible in patients with ulnar neuropathy. They present a case of a patient with ulnar neuropathy secondary to a ganglion cyst in guyon's canal.

     

    Maroukis et al. review the history of the clinical anatomy of Guyon's canal. They conclude that the three zone theory helped simplify the complex anatomy of ulnar nerve compression at Guyon's canal.

     

    Figure A (Shen et al) shows a T2 fat saturation MRI of a well circumscribed lesion (black arrow) with homogeneous fluid signal intensity at Guyon's canal compressing the ulnar nerve (white arrow). Figure B (Shen et al) shows a post contrast T1 fat saturation MRI showing rim enhancement consistent with a cyst (black arrow) and compression of the ulnar nerve (white arrow). Figure C shows an longitudinal ultrasound view of a anechoic well defined structure consistent with a cyst. Illustration A shows the areas of potential ulnar nerve compression in Guyon's canal. Illustration B shows a table with potential causes for compression at each zone and expected symptoms.

     

    Incorrect Answers:

    Answer 1: Imaging shows a well circumscribed homogeneous lesion consistent with a ganglion cyst, making a biopsy unnecessary.

    Answer 3: The patient has symptoms and EMG findings of peripheral nerve compression at the wrist.

    Answer 4: There is no fracture in the clinical history or imaging findings that may require hook of hamate excision.

    Answer 5: Cubital tunnel release is not indicated as the patients symptoms are localized distally at the level of Guyon's canal.

     

     

     

     

     

     

     

     

     

  105. A 20-year-old male presents to clinic for evaluation of right wrist pain. He fell playing flag football about 6 weeks ago. He initially had significant pain but since it slowly improved he did not seek immediate treatment. His improvement has now plateaued. Figures A and B are x-rays, and figures C and D select CT scan images of his right wrist. What is the best treatment option?

     

     

     

     

     

     

     

     

    1. Percutaneous screw fixation

    2. Open reduction internal fixation through a volar approach

    3. Open reduction internal fixation through a dorsal approach

    4. Open reduction internal fixation with bone grafting through a volar approach

    5. Open reduction internal fixation with bone grafting through a dorsal approach

    CorreCt answer: 4

     

    The patient presents with a displaced right scaphoid waist fracture with cyst formation. The best treatment would open reduction internal fixation (ORIF) with bone grafting through a volar approach.

     

    The surgical management of scaphoid fracture depends on location and characteristics of the fracture as well as time from injury. Displaced distal pole and waist fractures are typically approached from the volar side, especially if there is a humpback deformity; the proximal pole is more easily accessed from the dorsal side. Injuries with significant comminution or cyst formation due to extended time to treatment are often augmented with bone graft. There is controversy as to the use of vascularized bone graft in nonunion cases.

     

    Rettig et al. reported on fourteen patients undergoing acute surgical fixation for displaced scaphoid waist fractures. Thirteen patients united and regained functional wrist range of motion and grip strength. They advocate for early

    operative intervention in these fractures.

     

    Raskin et al. describe the utility of the dorsal approach for proximal pole scaphoid fractures. They report good fracture visualization and the ability to bone graft through the same incision with successful union in a majority of cases.

     

    Pinder et al. reviewed the literature on management of scaphoid nonunions. They found no difference in use of nonvascularized or vascularized bone graft, choice of approach, or use of Kirschner wires versus screw fixation.

     

    Figures A and B are postero-anterior lateral right wrist radiographs with a displaced scaphoid waist fracture and mild humpback deformity. Figures C and D are coronal and sagittal CT cuts, respectively, demonstrating cyst formation and better showing the humpback deformity.

     

    Incorrect Answers:

    Answer 1: An open approach is required to address this fracture

    Answer 2: Given the extent of cyst formation, ORIF with bone grafting is the better answer

    Answers 3 and 5: Scaphoid waist fractures with humpback deformity are best addressed through a volar approach

     

     

     

  106. A 53-year-old white male presents with contractures of his ring finger and lesions over the dorsum of his hand. On examination of the lesions, they are subcutaneous, solid, and firm lesions measuring about 5 mm in diameter. They are located over the dorsum of the PIP joints of his ring and long finger. They become more mobile while the joint is in neutral and less mobile when the joint is in flexion. He also has a 5 degree flexion contracture his ring finger MCP joint. Examination of his palm reveals a palpable cord over the volar ring finger. His neurovascular examination is normal. The appearance of the dorsum of his hand is seen in Figure A. What is the next most appropriate step in treatment?

     

     

     

    1. Collagenase injection and resection of dorsal finger lesions

    2. Collagenase injection without resection of dorsal finger lesions

    3. Observation and follow up

    4. Surgical resection/fasciectomy and resection of dorsal finger lesions

    5. Surgical resection/fasciectomy without resection of dorsal finger lesions

    CORRECT ANSWER: 3

    This patient has mild Dupuytren's disease with associated dorsal Dupuytren nodules, which may be observed.

     

    Dupuytren’s disease is a proliferative disorder characterized by fascial nodules and contractures of the hand. It is autosomal dominant with variable penetrance. It exhibits a 2:1 male to female ratio and is classically seen in Caucasian males of northern European descent. The main pathology of

    Dupuytren’s disease is excessive myofibroblast proliferation and altered collagen matrix composition lead to thickened and contracted palmar fascia. Surgical intervention is often indicated in cases of ≥30° of MCP contracture or any PIP contracture (usually >15°).

     

    Rayan et al report that dorsal Dupuytren's nodules are a subcutaneous, solid, firm, well-defined, tumor-like mass or a nodule 3 mm in diameter or larger, located over the dorsum of the PIP joint. It is seldom painful and becomes more mobile while the joint is in neutral position and less mobile during joint flexion.

    Black et al report that diseased tissue is referred to as nodules or cords. The Dupuytren nodule is a palpable subcutaneous lump that may be fixed to the skin. Cords are highly organized collagen structures arranged in parallel with a relatively hypocellular matrix. Cords are predominantly composed of collagen III while normal palmar fascia is predominantly collagen I.

    Figure A is a picture of a dorsal Dupuytren's nodule. Incorrect Answers:

    Answer 1: This patient has a mild contracture and no treatment is necessary

    at this time. The dorsal finger lesions should not be resected.

    Answer 2: This patient has a mild contracture and no treatment is necessary at this time.

    Answer 4: This patient has a mild contracture and no treatment is necessary at this time. The dorsal finger lesions should not be resected.

    Answer 5: This patient has a mild contracture and no treatment is necessary at this time.

     

     

     

  107. A 37-year-old man has a 2-year history of increasing right wrist pain that is worse at night and aggravated by activity. He denies systemic symptoms, history of trauma, or recent weight loss. On physical exam he has tenderness over the dorsal radiocarpal joint. Radiographs of the right wrist are shown in Figure A. Which of the following imaging studies would be most sensitive for determining the stage of this patient's underlying condition?

     

     

     

    1. Ultrasound

    2. Angiography

    3. CT scan of the wrist

    4. Clenched fist AP radiograph of wrist

    5. Bone scan of the wrist CORRECT ANSWER: 3

    The clinical presentation of dorsal radiocarpal wrist pain is suggestive of Kienbock’s disease. Figure A shows an AP radiograph of the right wrist with

    evidence of lunate sclerosis with no obvious collapse. The imaging study most sensitive for identifying early lunate collapse in Kienbock's disease is CT scanning of the wrist.

     

    Kienbock’s disease is defined by avascular necrosis of the lunate. It is classified into 4 stages under the Lichtman Classification. In stage 1, plain radiographs appear normal and magnetic resonance imaging is required for diagnosis. MRI is useful for detecting early disease when sclerosis is not evident on plain film radiographs. In stage 2, plain radiographs and/or CT scan images will show sclerosis of the lunate but no evidence of collapse. In stage 3, radiographs and/or CT scan images will show lunate collapse. For stage 4, radiographs show degenerative changes to the adjacent carpus and intercarpal joints.

     

    Imaeda et al. examined the use of MRI for the diagnosis and staging of Kienbock's disease. They found that MRI was most sensitive in detecting early focal loss of signal intensity in the lunate on T1-weighted images. This was a key diagnostic feature in early stages of Kienböck's disease when plain radiographs appear normal.

     

    Cross et al. reviewed the latest concepts for diagnosis, staging, and management of Keinbock's disease. They suggest that computed tomography (CT) or tomography will better characterize lunate necrosis and trabecular destruction once collapse or sclerosis has occurred in late stage disease.

     

    Illustration A is a collection of CT scanning images that show osteonecrosis of the lunate. The blue arrow shows lunate flattening and sclerosis. The red double arrow shows a loss of lunate height and the yellow shows fragmentation of the bone.

     

    Incorrect Answers:

    Answer 1: Ultrasound is not used in the staging of Kienbock's disease. Answer 3: Angiography would not be warranted in this scenario.

    Answer 4: A clenched fist AP radiograph of the wrist is used to evaluate

    widening of the scapholunate interval.

    Answer 5: A bone scan of the wrist is a non-specific test, which would likely be positive in almost all patients with chronic wrist pain.

     

     

     

     

     

     

  108. Each of the following are indications for microvascular replantation EXCEPT?

    1. Thumb amputation

    2. Index finger amputation in a child

    3. Through wrist amputation

    4. Long finger amputation through the proximal phalanx

    5. Mid-palm amputation of all four fingers

    CORRECT ANSWER: 4

    As reviewed by Soucacos, there are several major indications for single digit replantation: 1) Level of the amputation is distal to the insertion of FDS. 2) Amputations at the level of the distal phalanx. 3) Ring avulsion injuries involving both the dorsal and palmar skin and blood supply in an isolated finger, as long as FDS is intact. 4) Any amputation in a child. 5) Thumb amputation. Replantation of a single digit, which is amputated at the level of the proximal phalanx or at the PIP joint, particularly in avulsion or crush injury is contra-indicated. Soucacos also discusses appropriate surgical teams, transport, and other related issues surrounding a "transplant team."

     

     

  109. All of the following are predictive findings for correctly diagnosing carpal tunnel syndrome EXCEPT:

    1. Abnormal hand diagram

    2. Abnormal Semmes-Weinstein testing in wrist-neutral position

    3. Positive median nerve compression test (Durkan's sign)

    4. Presence of night pain

    5. Loss of small digit adduction (Wartenberg sign)

      CORRECT ANSWER: 5

      All of the listed physical exam findings, except for loss of small digit adduction (Wartenberg sign), has been found to be predictive for diagnosing carpal tunnel syndrome.

       

      Szabo et al in a Level 3 study used a regression model to analyze the most predictive factors for correctly diagnosing carpal tunnel syndrome (CTS). Their analysis found that with an abnormal hand diagram, abnormal sensibility by Semmes-Weinstein testing in wrist-neutral position, a positive Durkan's test, and night pain, the probability that carpal tunnel syndrome will be correctly diagnosed is 0.86. They found the tests with the highest sensitivity were Durkan's compression test (89%), Semmes-Weinstein testing after Phalen's maneuver (83%), and hand diagram scores (76%). Night pain was a sensitive symptom predictor (96%). The most specific tests were the hand diagram (76%) and Tinel's sign (71%). The authors concluded that the addition of electrodiagnostic tests did not increase the diagnostic power of the combination of these 4 clinical tests, and proceeding with surgical release is appropriate even if the EMG is normal.

       

      Wartenberg sign is persistent abduction and extension of the small digit when a patient is asked to adduct the digits and is seen in cubital tunnel syndrome, but not carpal tunnel syndrome.

       

      Illustration V demonstrate the Durkan's Compression test for carpal tunnel syndrome.

       

       

       

       

       

       

  110. Extrinsic imbalance from splinting a crushed hand with metacarpophalangeal joint extension causes what characteristic hand deformity?

    1. Distal interphalangeal joint extension

    2. Ulnar subluxation of the metacarpophalangeal joints

    3. Proximal interphalangeal joint extension

    4. Proximal interphalangeal joint flexion

    5. Swan-neck deformity CORRECT ANSWER: 4

    Failure to splint the hand in an intrinsic positive position leads to increased extrinsic finger flexor tension, leading the DIP and PIP joints to have an increasing flexion position. Illustration A and B show a clinical image and illustration of intrinsic minus hand.

     

    von Schroeder et al present a Level 5 review of hand crush injuries. They conclude that early diagnosis and treatment is critical, but the functional outcome is often poor with associated Volkmann's contracture.

     

     

     

     

     

     

     

     

     

  111. Axon regeneration almost always occurs following a Sunderland second-degree nerve injury because which anatomic structure is not injured?

    1. Epineurium

    2. Endoneurium

    3. Perineurium

    4. Myelin sheath

    5. Schwann cell CORRECT ANSWER: 2

    Following a Sunderland second-degree injury, axon regeneration is possible because the endoneurium is intact.

     

    There are two classification schemes for peripheral nerve injuries, which include the Seddon and the Sunderland systems. Under the Sunderland

    classification, a second-degree injury is considered a part of the axonotmesis spectrum. The endoneurium, perineurium and epineurium are still intact. This enables complete functional recovery.

     

    Lee et al. review the pathophysiology and evaluation of peripheral nerve injuries. They note that in Sunderland type two injuries, there is physiologic disruption of the axons. Because the endoneurium is still intact, axons are able to regenerate. This process takes months.

    Illustration A is a schematic of the various stages of peripheral nerve injury. Incorrect Answers

    Answers 1, 3: Although the epineurium and perineurium are intact in a

    Sunderland type 2 injury, axon regeneration is possible because of an intact endoneurium.

    Answers 4, 5: The myelin sheath and Schwann cells are disrupted in axonotmesis.

     

     

     

     

     

     

  112. A 29-year-old intravenous drug user undergoes irrigation and debridement of a ring finger abscess. After adequate eradication of the infection, he is left with the skin defect shown in Figure A. What is the most appropriate treatment at this time?

     

     

     

    1. Local woundcare and healing by secondary intention

    2. V-Y advancement flap

    3. Thenar flap

    4. Moberg flap

    5. Cross-finger flap CORRECT ANSWER: 5

    Based on the location of the lesion, a cross-finger flap would be most appropriate.

     

    Cross finger flaps are indicated in patients > 30 years of age when the lesion is a volar oblique finger tip lacerations or a volar proximal finger lesions. The advantage is it leads to less stiffness.

     

    Martin et al review the treatment options available for digit injuries. They report treatment of fingertip injuries is a continuous focus of controversy among hand and orthopaedic surgeons. Different treatment options have been described, depending on the affected segment and finger, type of lesion, gender and age of the patient, location, size, and depth of the defect.

     

    Fassler et al reviews the proper management of fingertip injuries discussing variables such as the severity of soft tissue loss and whether bone is exposed.

     

    Incorrect Answers:

    Answer 1: Secondary intention healing of this wound is inappropriate due to size and exposed tendon.

    Answer 2: V-Y advancement flaps are for dorsal injuries.

    Answer 3: Thenar flaps are good for getting more bulk for distal fingertip injuries.

    Answer 4: A Moberg flap is performed on the thumb. A cross-finger flap is a full-thickness flap useful for volar soft tissue loss distal to PIP.

     

     

  113. A 4-year-old boy sustains a flexor tendon laceration in Zone 2 of his 4th digit when he attempts to grab a knife. Optimal surgical management and postoperative rehabilitation consists of:

    1. 2 strand core suture technique and gentle active flexion and extension exercises with wrist in extension

    2. 2 strand core suture technique and cast immobilization for 8 weeks

    3. 4 strand core suture technique and gentle active flexion and extension exercises with wrist in extension

    4. 4 strand core suture technique and cast immobilization for 4 weeks

    5. 4 strand core suture technique and cast immobilization for 8 weeks

      CORRECT ANSWER: 4

      4 strand core suture technique and cast immobilization for 4 weeks is the preferred postoperative rehabiltation in a 4 year old child.

       

      Ordinarily, adult flexor tendon repair postoperative rehab protocols call for early light active digital flexion with wrist in gentle flexion as long as the tendon has been repaired with a 4 or 6 strand core suture technique and strong epitendinous suture. However, this method cannot succeed without the cooperation of a mature and motivated patient. Children or the mentally disabled are often lacking some of these prerequisites. Therefore, a flexor tendon repair in a child should be treated like a flexor tendon repair with interposed graft in an adult. Immobilization for a minimum of 3 – 4 weeks with a posterior molded plaster splint or cast from the tips of the fingers to just above the elbow. Wrist is flexed 35 degrees, MCPs flexed 60 – 70 degrees and IP joints relaxed in extension. Active motion can be started after the cast is removed at 4 weeks.

       

       

       

  114. A 45-year-old male sustained a fall onto his right wrist 2 weeks ago. A radiograph is shown in figure A. What joint is first affected if left untreated with subsequent development of a SLAC (scapholunate advanced collapse) wrist?

 

 

 

  1. Capitolunate joint

  2. Radioscaphoid

  3. Radioulnar

  4. Radiolunate

  5. STT (scaphotrapezotrapezoidal)

CORRECT ANSWER: 2

The clinical presentation is consistent with a SLAC wrist. The radioscaphoid joint is the first to be affected in this process.

 

The radiographs of the right wrist demonstrate a scapholunate dissociation, as evidenced by an increased scapholunate joint space, referred to as scapholunate diastasis (abnormal when the gap is greater than 2 mm and increased from the opposite extremity and other intercarpal spaces).

 

If left untreated, the wrist may progress to a "SLAC" wrist, as originally described by Watson and Ballet in 1984, which is the most common form of wrist arthritis. The repetitive sequence of degenerative changes is based on and caused by articular alignment problems between the scaphoid, the lunate and the radius.

Kuo et al. review the stages of SLAC wrist. They report stage I SLAC wrist involves changes limited to an area of abnormal contact between the abnormally rotated scaphoid and the radial styloid. In stage II the remaining radioscaphoid joint is affected, as persistent abnormal load transfer and shear across the cartilaginous surfaces leads to degeneration of the proximal scaphoid facet. In stage III, the dorsally translated capitate migrates proximally into the widened scapholunate interval, and degenerative changes occur at the capitolunate joint. The relative congruency of the radiolunate joint in all positions of lunate rotation due to the spherical shape of the lunate facet preserves this articulation, and at all stages of SLAC wrist the radiolunate joint is not involved. The lunate is congruently loaded in every position and, thus, highly resistant to degenerative changes.

 

Illustration A below shows the stages of involvement in the SLAC wrist.